首页 2009-2013年考研英语历年真题集含答案解析

2009-2013年考研英语历年真题集含答案解析

举报
开通vip

2009-2013年考研英语历年真题集含答案解析2009-2013年考研英语历年真题集含答案解析 2013年全国硕士研究生入学统一考试英语试题 Section ? Use of English Directions: Read the following text. Choose the best word(s) for each numbered blank and mark A, B, C or D on ANSWER SHEET 1. (10 points) People are, on the whole, poor at considerin...

2009-2013年考研英语历年真题集含答案解析
2009-2013年考研英语历年真 快递公司问题件快递公司问题件货款处理关于圆的周长面积重点题型关于解方程组的题及答案关于南海问题 集含答案解析 2013年全国硕士研究生入学统一考试英语试题 Section ? Use of English Directions: Read the following text. Choose the best word(s) for each numbered blank and mark A, B, C or D on ANSWER SHEET 1. (10 points) People are, on the whole, poor at considering background information when making individual decisions. At first glance this might seem like a strength that 1 the ability to make judgments which are unbiased by 2 factors. But Dr. Uri Simonsohn speculated that an inability to consider the big 3 was leading decision-makers to be biased by the daily samples of information they were working with. 4 , he theorised that a judge 5 of appearing too soft 6 crime might be more likely to send someone to prison 7 he had already sentenced five or six other defendants only to probation on that day. To 8 this idea, he turned to the university-admissions process. In theory, the 9 of an applicant should not depend on the few others 10 randomly for interview during the same day, but Dr Simonsohn suspected the truth was 11 . He studied the results of 9,323 MBA interviews, 12 by 31 admissions officers. The interviewers had 13 applicants on a scale of one to five. This scale 14 numerous factors into consideration. The scores were 15 used in conjunction with an applicant‘s score on the Graduate Management Admission Test, or GMAT, a standardised exam which is 16 out of 800 points, to make a decision on whether to accept him or her. Dr Simonsohn found if the score of the previous candidate in a daily series of interviewees was 0.75 points or more higher than that of the one 17 that, then the score for the next applicant would 18 by an average of 0.075 points. This might sound small, but to 19 the effects of such a decrease a candidate would need 30 more GMAT points than would otherwise have been 20 . 1.[A] grant [B] submits [C] transmits [D] delivers 2.[A] minor [B]objective [C] crucial [D] external 3.[A] issue [B] vision [C] picture [D] moment 4.[A] For example [B] On average [C] In principle[D] Above all 5.[A] fond [B]fearful [C] capable [D] thoughtless 6.[A] in [B] on [C] to [D] for 7.[A] if [B]until [C] though [D] unless 8.[A] promote [B]emphasize [C] share [D] test 9.[A] decision [B] quality [C] status [D] success 10.[A] chosen [B]stupid [C]found [D] identified 11.[A] exceptional [B] defensible [C] replaceable [D] otherwise 12.[A] inspired [B]expressed [C] conducted [D] secured 13.[A] assigned [B]rated [C] matched [D] arranged 14.[A] put [B]got [C]gave [D] took - 1 - 15.[A]instead [B]then [C] ever [D] rather 16.[A]selected [B]passed [C] marked [D] introduced 17.[A]before [B] after [C] above [D] below 18.[A] jump [B] float [C] drop [D] fluctuate 19.[A]achieve [B]undo [C] maintain [D]disregard 20. [A] promising [B] possible [C] necessary [D] helpful Section ? Reading Comprehension Part A Directions:Read the following four texts. Answer the questions after each text by choosing A, B, C or D. Mark your answers on ANSWER SHEET 1. (40 points) Text 1 In the 2006 film version of The Devil Wears Prada, Miranda Priestly, played by Meryl Streep, scold her unattractive assistant for imagining that high fashion doesn‘t affect her. Priestly explains how the deep blue color of the assistant‘s sweater descended over the years from fashion shows to department stores and to the bargain bin in which the poor girl doubtless found her garment. This top-down conception of the fashion business couldn‘t be more out of date or at odds with feverish world described in Overdressed, Elizabeth Cline‘s three-year indictment of ,fast fashion‖. In the last decades or so, advances in technology have allowed mass-market labels such as Zara, H&M, and Uniqlo to react to trends more quickly and anticipate demand more precisely. Quckier turnrounds mean less wasted inventory, more frequent releases, and more profit. Those labels encourage style-conscious consumers to see clothes as disposal—— meant to last only a wash or two, although they don‘t advertise that——and to renew their wardrobe every few weeks. By offering on-trend items at dirt-cheap prices, Cline argues, these brands have hijacked fashion cycles, shaking all industry long accustomed to a seasonal pace. The victims of this revolution, of course, are not limited to designers. For H&M to offer a 5.95 knit miniskirt in all its 2300-plus stores around the world, it must rely on low-wage, overseas labor, order in volumes that strain natural resources, and use massive amount of harmful chemicals. Overdressed is the fashion world‘s answer to consumer activist bestsellers like Michael Pollan‘s The Omnivore‘s Dilemma. Mass-produced clothing, like fast food, fills a hunger and need, yet is non-durable, and wasteful,‖ Cline argues, Americans, she finds, buy roughly 20 billion garments a year——about 64 items per person——and no matter how much they give away, this excess leads to waste. Towards the end of Overdressed, Cline introduced her ideal, a Brooklyn woman named SKB, who, since 2008 has make all of her own clothes——and beautifully. But as Cline is the first to note, it took Beaumont decades to perfect her craft; her example, can‘t be knocked off. Though several fast-fashion companies have made efforts to curb their impact on labor and the environment——including H&M, with its green Conscious Collection Line——Cline believes - 2 - lasting-change can only be effected by the customer. She exhibits the idealism common to many advocates of sustainability, be it in food or in energy. Vanity is a constant; people will only start shopping more sustainably when they can‘t afford to it. 21. Priestly criticizes her assistant for her [A] poor bargaining skill.[B] insensitivity to fashion. [C] obsession with high fashion.[D]lack of imagination. 22. According to Cline, mass-maket labels urge consumers to [A] combat unnecessary waste. [B] shut out the feverish fashion world. [C] resist the influence of advertisements. [D] shop for their garments more frequently. 23. The word ,indictment‖ (Line 3, Para.2) is closest in meaning to [A] accusation. [B] enthusiasm. [C] indifference. [D] tolerance. 24. Which of the following can be inferred from the lase paragraph? [A] Vanity has more often been found in idealists. [B] The fast-fashion industry ignores sustainability. [C] People are more interested in unaffordable garments. [D] Pricing is vital to environment-friendly purchasing. 25. What is the subject of the text? [A] Satire on an extravagant lifestyle. [B] Challenge to a high-fashion myth. [C] Criticism of the fast-fashion industry. [D] Exposure of a mass-market secret. Text 2 An old saying has it that half of all advertising budgets are wasted-the trouble is, no one knows which half . In the internet age, at least in theory ,this fraction can be much reduced . By watching what people search for, click on and say online, companies can aim ,behavioural‖ ads at those most likely to buy. In the past couple of weeks a quarrel has illustrated the value to advertisers of such fine-grained information: Should advertisers assume that people are happy to be tracked and sent behavioural ads? Or should they have explicit permission? In December 2010 America‘s Federal Trade Cornmission (FTC) proposed adding a "do not track "(DNT) option to internet browsers ,so that users could tell adwertisers that they did not want to be followed .Microsoft‘s Internet Explorer and Apple‘s Safari both offer DNT ;Google‘s Chrome is due to do so this year. In February the FTC and Digltal Adwertising Alliance (DAA) agreed that the industry would get cracking on responging to DNT requests. On May 31st Microsoft Set off the row: It said that Internet Explorer 10,the version due to appear windows 8, would have DNT as a default. It is not yet clear how advertisers will respond. Geting a DNT signal does not oblige anyone to stop tracking, although some companies have promised to do so. Unable to tell whether someone really objects to behavioural ads or whether they are sticking with Microsoft‘s default, some may ignore a DNT signal and press on anyway. - 3 - Also unclear is why Microsoft has gone it alone. Atter all, it has an ad business too, which it says will comply with DNT requests, though it is still working out how. If it is trying to upset Google, which relies almost wholly on default will become the norm. DNT does not seem an obviously huge selling point for windows 8-though the firm has compared some of its other products favourably with Google‘s on that count before. Brendon Lynch, Microsoft‘s chief privacy officer, bloggde:"we believe consumers should have more control." Could it really be that simple? 26. It is suggested in paragraph 1 that ,behavioural‖ ads help advertisers to: [A] ease competition among themselves [B] lower their operational costs [C] avoid complaints from consumers [D]provide better online services 27. ,The industry‖ (Line 6,Para.3) refers to: [A] online advertisers [B] e-commerce conductors [C] digital information analysis [D]internet browser developers 28. Bob Liodice holds that setting DNT as a default [A] many cut the number of junk ads [B] fails to affect the ad industry [C] will not benefit consumers [D]goes against human nature 29. which of the following is ture according to Paragraph.6? [A] DNT may not serve its intended purpose [B] Advertisers are willing to implement DNT [C] DNT is losing its popularity among consumers [D] Advertisers are obliged to offer behavioural ads 30. The author‘s attitude towards what Brendon Lynch said in his blog is one of: [A] indulgence [B] understanding [C] appreciaction [D] skepticism Text 3 Up until a few decades ago, our visions of the future were largely - though by no means uniformly - glowingly positive. Science and technology would cure all the ills of humanity, leading to lives of fulfillment and opportunity for all. Now utopia has grown unfashionable, as we have gained a deeper appreciation of the range of threats facing us, from asteroid strike to epidemic flu and to climate change. You might even be tempted to assume that humanity has little future to look forward to. But such gloominess is misplaced. The fossil record shows that many species have endured for millions of years - so why shouldn‘t we? Take a broader look at our species‘ place in the universe, and it becomes clear that we have an excellent chance of surviving for tens, if not hundreds, of thousands of years . Look up Homo sapiens in the "Red List" of threatened species of the International Union for the Conversation of Nature (IUCN) ,and you will read: "Listed as Least Concern as the species is very widely distributed, adaptable, currently increasing, and there are no major threats resulting in an overall population decline." - 4 - So what does our deep future hold? A growing number of researchers and organisations are now thinking seriously about that question. For example, the Long Now Foundation has its flagship project a medical clock that is designed to still be marking time thousands of years hence . Perhaps willfully , it may be easier to think about such lengthy timescales than about the more immediate future. The potential evolution of today‘s technology, and its social consequences, is dazzlingly complicated, and it‘s perhaps best left to science fiction writers and futurologists to explore the many possibilities we can envisage. That‘s one reason why we have launched Arc, a new publication dedicated to the near future. But take a longer view and there is a surprising amount that we can say with considerable assurance. As so often, the past holds the key to the future: we have now identified enough of the long-term patterns shaping the history of the planet, and our species, to make evidence-based forecasts about the situations in which our descendants will find themselves. This long perspective makes the pessimistic view of our prospects seem more likely to be a passing fad. To be sure, the future is not all rosy. But we are now knowledgeable enough to reduce many of the risks that threatened the existence of earlier humans, and to improve the lot of those to come. 31. Our vision of the future used to be inspired by [A] our desire for lives of fulfillment [B] our faith in science and technology [C] our awareness of potential risks [D] our belief in equal opportunity 32. The IUCN‘s ,Red List‖ suggest that human being are [A] a sustained species [B] a threaten to the environment [C] the world‘s dominant power [D] a misplaced race 33. Which of the following is true according to Paragraph 5? [A] Arc helps limit the scope of futurological studies. [B] Technology offers solutions to social problem. [C] The interest in science fiction is on the rise. [D] Our Immediate future is hard to conceive. 34. To ensure the future of mankind, it is crucial to [A] explore our planet‘s abundant resources [B] adopt an optimistic view of the world [C] draw on our experience from the past [D] curb our ambition to reshape history 35. Which of the following would be the best title for the text? [A] Uncertainty about Our Future [B] Evolution of the Human Species [C] The Ever-bright Prospects of Mankind [D] Science, Technology and Humanity Text 4 On a five to three vote, the Supreme Court knocked out much of Arizona‘s immigration law Monday-a modest policy victory for the Obama Administration. But on the more important matter of the Constitution,the decision was an 8-0 defeat for the Administration‘s effort to upset the balance of power between the federal government and the states. - 5 - In Arizona v. United States, the majority overturned three of the four contested provisions of Arizona‘s controversial plan to have state and local police enforce federal immigration law. The Constitutional principles that Washington alone has the power to ,establish a uniform Rule of Naturalization ‖and that federal laws precede state laws are noncontroversial . Arizona had attempted to fashion state policies that ran parallel to the existing federal ones. Justice Anthony Kennedy, joined by Chief Justice John Roberts and the Court‘s liberals, ruled that the state flew too close to the federal sun. On the overturned provisions the majority held the congress had deliberately ,occupied the field‖ and Arizona had thus intruded on the federal‘s privileged powers. However,the Justices said that Arizona police would be allowed to verify the legal status of people who come in contact with law enforcement.That‘s because Congress has always envisioned joint federal-state immigration enforcement and explicitly encourages state officers to share information and cooperate with federal colleagues. Two of the three objecting Justice-Samuel Alito and Clarence Thomas-agreed with this Constitutional logic but disagreed about which Arizona rules conflicted with the federal statute.The only major objection came from Justice Antonin Scalia,who offered an even more robust defense of state privileges going back to the alien and Sedition Acts. The 8-0 objection to President Obama turns on what Justice Samuel Alito describes in his objection as ,a shocking assertion assertion of federal executive power‖.The White House argued that Arizona‘s laws conflicted with its enforcement priorities,even if state laws complied with federal statutes to the letter.In effect, the White House claimed that it could invalidate any otherwise legitimate state law that it disagrees with . Some powers do belong exclusively to the federal government, and control of citizenship and the borders is among them. But if Congress wanted to prevent states from using their own resources to check immigration status, it could. It never did so. The administration was in essence asserting that because it didn‘t want to carry out Congress‘s immigration wishes, no state should be allowed to do so either. Every Justice rightly rejected this remarkable claim. 36. Three provisions of Arizona‘s plan were overturned because they [A] deprived the federal police of Constitutional powers. [B] disturbed the power balance between different states. [C] overstepped the authority of federal immigration law. [D] contradicted both the federal and state policies. 37. On which of the following did the Justices agree, according to Paragraph4? [A] Federal officers‘ duty to withhold immigrants‘information. [B] States’ independence from federal immigration law. [C] States‘ legitimate role in immigration enforcement. [D] Congress‘s intervention in immigration enforcement. 38. It can be inferred from Paragraph 5 that the Alien and Sedition Acts - 6 - [A] violated the Constitution. [B] undermined the states‘ interests. [C] supported the federal statute. [D] stood in favor of the states. 39. The White House claims that its power of enforcement [A] outweighs that held by the states. [B] is dependent on the states‘ support. [C] is established by federal statutes. [D] rarely goes against state laws. 40. What can be learned from the last paragraph? [A] Immigration issues are usually decided by Congress. [B] Justices intended to check the power of the Administrstion. [C] Justices wanted to strengthen its coordination with Congress. [D] The Administration is dominant over immigration issues. Part B Directions: In the following article, some sentences have been removed. For Questions 41-45, choose the most suitable one from the list A-G to fit into each of the numbered blank. There are two extra choices, which do not fit in any of the gaps. Mark your answers on ANSWER SHEET 1. (10 points) The social sciences are flourishing. As of 2005,there were almost half a million professional social scientists from all fields in the world, working both inside and outside academia. According to the World Social Science Report 2010,the number of social-science students worldwide has swollen by about 11% every year since 2000. Yet this enormous resource in not contributing enough to today‘s global challenges including climate change, security, sustainable development and health.(41)______Humanity has the necessary agro-technological tools to eradicate hunger , from genetically engineered crops to artificial fertilizers . Here , too, the problems are social: the organization and distribution of food, wealth and prosperity. (42)____This is a shame—the community should be grasping the opportunity to raise its influence in the real world. To paraphrase the great social scientist Joseph Schumpeter: there is no radical innovation without creative destruction . Today ,the social sciences are largely focused on disciplinary problems and internal scholarly debates, rather than on topics with external impact. Analyses reveal that the number of papers including the keywords ,environmental changed‖ or ,climate change‖ have increased rapidly since 2004,(43)____ When social scientists do tackle practical issues ,their scope is often local:Belgium is interested mainly in the effects of poverty on Belgium for example .And whether the community‘s work contributes much to an overall accumulation of knowledge is doubtful. The problem is not necessarily the amount of available funding (44)____this is an adequate amount so long as it is aimed in the right direction. Social scientists who complain about a lack of funding should not expect more in today‘s economic climate. - 7 - The trick is to direct these funds better.The European Union Framework funding programs have long had a category specifically targeted at social scientists.This year,it was proposed that system be changed:Horizon 2020,a new program to be enacted in 2014,would not have such a category ,This has resulted in protests from social scientists.But the intention is not to neglect social science ; rather ,the complete opposite.(45)____That should create more collaborative endeavors and help to develop projects aimed directly at solving global problems. [A] It could be that we are evolving two communities of social scientists: one that is discipline-oriented and publishing in highly specialized journals, and one that is problem-oriented and publishing elsewhere, such as policy briefs. [B] However,the numbers are still small:in 2010,about 1,600 of the100,000 social-sciences papers published globally included one of these Keywords. [C] the idea is to force social to integrate their work with other categories, including health and demographic change food security, marine research and the bio-economy, clear, efficient energy; and inclusive, innovative and secure societies. [D] the solution is to change the mindset of the academic community, and what it considers to be its main goal. Global challenges and social innovation ought to receive much more attention from scientists, especially the young ones. [E] These issues all have root causes in human behavior . all require behavioral change and social innovations , as well as technological development . Stemming climate change , for example , is as much about changing consumption patterns and promoting tax acceptance as it is about developing clean energy. [F] Despite these factors , many social scientists seem reluctant to tackle such problems . And in Europe , some are up in arms over a proposal to drop a specific funding category for social-science research and to integrate it within cross-cutting topics of sustainable development . [G] During the late 1990s , national spending on social sciences and the humanities as a percentage of all research and development funds-including government, higher education, non-profit and corporate -varied from around 4% to 25%; in most European nations , it is about 15%. Part B: (10 points) Section III Translation 46. Directions: Translate the following text from English to Chinese. Write your translation on ANSWER SHEET2. (10 points) Directions: Read the following text carefully and then translate the underlined segments into Chinese. Your translation should be written clearly on ANSWER SHEET 2. (10 points) It is speculated that gardens arise from a basic need in the individuals who made them: the need for creative expression. There is no doubt that gardens evidence an impossible urge to create, express, fashion, and beautify and that self-expression is a basic human urge; (46) Yet when one looks at the photographs of - 8 - the garden created by the homeless, it strikes one that , for all their diversity of styles, these gardens speak os various other fundamental urges, beyond that of decoration and creative expression. One of these urges had to do with creating a state of peace in the midst of turbulence, a ,still point of the turning world,‖ to borrow a phrase from T. S. Eliot. (47)A sacred place of peace, however crude it may be, is a distinctly human need, as opposed to shelter, which is a distinctly animal need. This distinction is so much so that where the latter is lacking, as it is for these unlikely gardens, the former becomes all the more urgent. Composure is a state of mind made possible by the structuring of one‘s relation to one‘s environment. (48) The gardens of the homeless which are in effect homeless gardens introduce from into an urban environment where it either didn‘t exist or was not discernible as such. In so doing they give composure to a segment of the inarticulate environment in which they take their stand. Another urge or need that these gardens appear to respond to, or to arise from is so intrinsic that we are barely ever conscious of its abiding claims on us. When we are deprived of green, of plants, of trees, (49)most of us give into a demoralization of spirit which we usually blame on some psychological conditions, until one day we find ourselves in garden and feel the expression vanish as if by magic. In most of the homeless gardens of New York City the actual cultivation of plants is unfeasible, yet even so the compositions often seem to represent attempts to call arrangement of materials, an institution of colors, small pool of water, and a frequent presence of petals or leaves as well as of stuffed animals. On display here are various fantasy elements whose reference, at some basic level, seems to be the natural world. (50)It is this implicit or explicit reference to nature that fully justifies the use of word garden though in a ,liberated‖ sense, to describe these synthetic constructions. In them we can see biophilia- a yearning for contact with nonhuman life-assuming uncanny representational forms. Section III Writing Party A 51 Directions: Write an e-mail of about 100 words to a foreign teacher in your college inviting him/her to be a judge for the upcoming English speech contest. You should include the details you think necessary. You should write neatly on the ANSWER SHEET. Do not sign your own name at the end of the e-mail. Use ,Li Ming‖ instead. Do not write the address. (10 points) Part B: (20 points) Part B 52 Directions: Write an essay of about 160 – 200 words based on the following drawing. In your essay, you should (1) describe the drawing briefly, - 9 - (2) interpret its intended meaning, and (3) give your comments. You should write neatly on the ANSWER SHEET. (20 points) 2013年考研英语真题答案 1.【答案】A 【解析】第一句提到―总体而言,当人们自己做决定时,并不擅长考虑背景 信 息。‖第二句顺接上文,―乍一看这是一种优势‖,that引起定语从句,这种优势使 人们具有一种能力,即能够做出不受外界因素影响的不带偏见的决定。B选项 submit ―服从,提交‖,不能与ability连用,C选项transmit ―传输,发射‖,也不 能与ability 搭配,D选项deliver ―传递‖,同样不能与ability搭配。A, C, D无论 从搭配上还是意思上都不合适。A选项grant本身具有赋予,授予的意思。故答 案选A。 - 10 - 2.【答案】D 【解析】external外部因素和上文的background information同义复现,不考虑 背景信息,不受外界因素影响。A选项minor 次要的,B选项objective 客观的, C选项crucial 残酷的,D选项external 外部的,故答案选D。 3.【答案】C 【解析】第三题本句but引起句意转折。―但是XX推测不考虑大局会导致决策 者被日常接触的信息影响而带有偏见。‖首先注意到空前面有定冠词the,指代上文信息,即不考虑背景信息、不考虑大环境。而大局,大环境的表达,此处选择picture是最贴切的。A选项 issue 问题,B选项vision 想象力,美景都不合适,故答案选C。 4.【答案】A 【解析】通读后面的句子,提到了法官与被告,这明显是生活当中的一个具体的实例,故答案选A。而B选项 on average ―平均,通常‖,出现的话,周围往往应该要出现数字。C选项in principle―大体上,原则上‖,后面需要出现的是总结性的话语,D选项above all―首先‖ 是用来列举条目,将A,B,D排除。 5.【答案】B 【解析】从句意上来看―例如,他们提出理论,认为法官不敢在罪行面前表现得太软弱,如果当天已经宣判五六名被告执行缓刑,那么他很有可能将下一个人送入监狱。A选项fond of 喜欢,B选项 fear of 惧怕,C选项capable of 有能力,D选项thoughtless of 考虑不周,故答案选B。 6.【答案】B 【解析】根据句 - 11 - 来看,说―这个等级考虑了几种因素…‖,是对上文评级的进一步解释,也没有问题。 15.【答案】B 【解析】还是承接上文讲到的评级得分,后半句讲到的是(平时学校等级)考试得分,再结合中间conjunction一词(联接),可以推断为then(具有承接之意),因此为正确答案,而A和D为同一意义和用法(代替),与conjunction相冲突,C说不通,故也为错误选项。 16.【答案】C 【解析】本题出在一个非限定性定语从句上,先行词为a standardized exam, 后半句是800分,考试和分数之间首选marked,选项B通过 具有一定的干扰性,但注意主语是考试,所以正确选项为C。A为无关选项。 17.【答案】A 【解析】本句属于比较级,对比的是几个面试者的分数,C、D是空间上的上下,而这里缺的时间上的先后,故排除C和D,B是―之后‖,不符合语言先后逻辑,故正确答案为A. 18.【答案】C 【解析】解本题需往下看,to…the effects of such a decrease, 由此可以判断接下来那个应聘者的分数是出现了下降,故直击正确选项C. 19.【答案】B 【解析】该句为不定式作主语,―(面试考官可能给)更低的分数‖所带来的影响,可以推断,是消除或是抵消这种不利结果,应聘者需要在GMAT中多拿30分,A ―达到‖,C ―保持‖D―漠视‖明显不符,故A―消除‖为正确答案。 20.【答案】C 【解析】该题难度较大,需把句意弄懂,也就是―这30分是比…所多的‖建议把四个选项分别代入空格处,A有前途的,B可能的,C必须的,D有帮助的,对比之后,只有C最合逻辑,也就是说―这30分是比原本该考试所必须要求的 多出来的,即另外多拿本不需要的30分才能平衡前面的更低的分数‖,故C为 正确选项。 21.【答案】B (insensitivity to fashion) 【解析】事实细节题。根据题干,首先定位到首段。由文章第一句后半句―…scolds her unattractive assistant for imagining that high fashion doesn?t affect her.‖意思是: ―……批评她没有魅力的助理,因为助理认为高级时尚对她的生活影响不大‖。可 知criticize是对scolds的同义替换,B项中的―insensitivity to fashion‖是―imagining that high fashion doesn?t affect her.‖的同义替换。所以B项为正确答案。 A项在文中并未提及,属于无中生有。C项和D项是对文章第一句的曲解。 22.【答案】D (shop for their garments more frequently) 【解析】事实细节题。根据题干,首先定位到第二段。由倒数第二句―these labels encourage style-conscious consumers to see clothes as disposable, ……, and to renew their wardrobe every few weeks.‖意思是―这些商标(畅销商标)促使有时尚意识的 消费者将服装看成是用完就可以丢弃的,……,并且每周更新他们的衣橱。‖D 选项 ―shop for their garments more frequently‖的意思是―更加频繁地购买服装‖, 正好是―renew their wardrobe every few weeks‖的同义替换。 A,B,C项均属于无中生有项。 23.【答案】A (accusation) 【解析】词义题。题干中需要猜测词义的单词出现在第二段的第一句―……the feverish world described in Overdressed, Elizabeth Cline?s three-year indictment of „fast fashion?‖。再结合选项可知,―indictment‖是Elizabeth Cline对―快时尚‖的一 种态度。因此,解答此题的关键在于联系上下文语境,找到Elizabeth Cline对―快 时尚‖的态度。由第二段最后一句―By offering on-trend items at dirt-cheap prices, Cline argues, these brands have hijacked fashion cycles, shaking an industry long accustomed to a seasonal pace.‖,意思是―Cline说,通过以特别低的价格销售潮流 物品,这些品牌破坏了潮流周期,动摇了这个长久以来习惯于季节周期的产业‖。 由―hijack‖和―shaking‖可知,Cline对―快时尚‖应该是持否定态度的,所以选项 A―accusation (谴责)‖是正确选项。 24.【答案】D (pricing is vital to environmental-friendly purchasing) 【解析】推理判断题。根据题干,可定位到最后一段。解题关键在于―Vanity is a constant; people will only start shopping more sustainably when they can?t afford not to‖,意思是―每个人都很虚荣,这很常见。但消费者付不起太多东西的时候, 他们才会以更加可持续的方式去购物。‖这句的关键词是―afford‖和―shop more sustainably‖, - 12 - 对应于D项中的―pricing‖和―environmental-friendly purchasing‖。A项对于本段 的曲解。B项说的是―忽视环境的可持续发展‖,与文中―several fast-fashion companies have made efforts to curb their impact on labor and the environment…‖(一 些时尚服饰公司已经做出努力减少对劳动力和环境)意思相悖。C项文中未提及。 25.【答案】C (criticism of the fast-fashion industry) 【解析】主旨大意题。此题考查对全文主旨大意的准确归纳。从整个文章脉络 来看,文章的第一段用事例引入,第二段讲到文章的主题―快时尚‖,并指出它破 坏了时尚周期,动摇了时尚产业。第三四段指出―快时尚‖这种变革的弊端,比如: 给自然资源造成压力、使用大量有害的化学物质、浪费现象。最后两段提到针对 ―快时尚‖的不良影响,可以采取的解决办法。由此可知,C项统领全文,为正确 答案。 A, B,D项都不是文章所论述的中心主题。 26.【答案】C (lower their operational costs) 【解析】事实细节题。根据题干,首先定位到首段。这段的大意是广告经费的 一半都浪费掉了,但是通过―behavioral ads‖可以追踪购买者的搜索习惯和 评价 LEC评价法下载LEC评价法下载评价量规免费下载学院评价表文档下载学院评价表文档下载 , 使得广告更有针对性,从而降低预算成本,也就是―this fraction can be much reduced‖。 A、B和C选项文中并未提及,属于无中生有。 27.【答案】D (internet browser developers) 【解析】词义句意题。the industry在语篇中是指代前面的出现 - 13 - 考眼前的未来相比,对如此之长的时间跨度进行思考似乎更为容易‖,反过来 思考也就是说,眼前的未来更难思考,符合D选项含义―我们眼前的未来很难去 设想‖。A选项―Arc 帮助缩小了未来学研究的范围‖,该选项在文中出现在该段 的最后一句话―这就是为什么我们可以发行Arc这样一个致力于研究近期未来的 全新出版物‖,和题目含义有很大出入,故排除。B选项―技术为社会问题提供了 解决 方法 快递客服问题件处理详细方法山木方法pdf计算方法pdf华与华方法下载八字理论方法下载 ‖,段中并未提到。C选项―对科幻小说的兴趣与日俱增‖,该段中虽在 第二句提到科幻小说家,但并未提及对科幻小说的兴趣,故排除。正确答案为D 项―Our immediate future is hard to conceive‖。 34.【答案】[C] draw on our experience from the past 【解析】段落细节题。由本题的题干可以锁定本题的答案在倒数第二自然段第 二句,―As so often , the past holds the key to the future . ‖此题就是考查对这句话的 理解,― 未来是掌握过去的关键。‖由此确定正确答案为B。选项A 、C、D 与 题干无关,在原文中无直接体现,也不能归纳得出,故排除 。 35.【答案】[C] The Ever-bright Prospects of Mankind 【解析】全文的主旨题。考查考生对全文主题的把握。通观全文,我们可发现 作者对未来是十分看好的,尤其在文章最后一段最后一句―But we are now knowledgeable enough to reduce many of the risks that threatened the existence of earlier humans, and to improve the lot of those to come. ‖作者直接借助这句话重申 主题。由此,本题正确答案应既包含―未来‖,也应能体现出作者对未来的态度。 确定选项C为正确答案。选项A错在无中生有,全文当中对于未来,并无体现 出对于其的不确定; 36.【答案】 overstepped the authority of federal immigration law. 【解析】事实细节题。principles that federal laws precede state laws are noncontroversial are noncontroversial.说明联邦法律高于州的法律是无可争辩的。 答案选项they―overstepped the authority of federal immigration law.‖---他们(亚利桑 那州的法案)逾越了联邦法案。就是对文中这句话的反义改写。Overstep 为同义 替换原文中的intrude, authority 同义替换了privileged powers. 属于同义置换。(考|研教育网整理) 37.【答案】States? legitimate role in immigration enforcement.州政府在移民法案 实施中的合法地位。 【解析】第四段主要说明了,州警察依然可以核实移民的法律地位。国会设想joint federal-state immigration enforcement联合实施移民法案。同时,encourages state officers to share information and cooperate with federal colleagues.国会鼓励州警察与联邦同事分享信息以及相互合作。其他选项的withhold,independence,intervention文中也没有提到。属于过度推断。 38.【答案】(Stood in favor of the states) 【解析】第五段最后一句:唯一的最主要的反对来自法官Antonino Scalia, 这个法官―defense‖ 是支持州的权利的,―going back to‖可追溯到Alien and Sedition Acts,证明这个法案是支持州的权利的。 39.【答案】(outweights that held by the states.) 联邦政府的权利大过州的实施权利 【解析】第六段The White House 认为亚利桑那州的法律跟白宫的法律实施权利冲突。In effect后面表达的是重点:如果这些州的法律跟它有冲突的话,白宫声明它有权利宣布其它州的法律无效。 40.【答案】(The Administration is dominant over immigration issues.) (政府在移民问题上占据着主导地位) 【解析】本段第一句话,联邦政府确实有一些exclusively(专门地)权利,比如控制居民以及边界。这就暗示了移民问题上,政府当局是具有主导权利的。 Part B 41.【答案】G (These issues all have root causes in human behavior...) 【解析】此题可以通过上下文的衔接和代词指代来确定答案。空格前一句谈到―这种巨大的能源不是当今全球问题的主要影响因素,这些问题包括气候变化、安全、可持续发展和健康问题‖,空后谈到―人类有必要的农业技术工具来消除饥饿‖,空格处应该填入的选项可以连接前后句的内容,既包括谈及到全球问题,又谈及到解决问题的选项只有G项。该项首句提到的these issues即指代空前所提及的全球问题,以及该项第二句的climate change举例说明即是空前所列出的问题之一,并且该项提及解决气候变化的问题,很好的启示了下文。 - 14 - 42.【答案】C (Despite these factors...) 【解析】本题可以通过连贯性原则和代词指代来确定答案。上段末句提到―问题也带有社会因素:对食物的组织和分配,财产和财富‖,空格后谈到―这是一种耻辱,社会应该抓住机会提升它在真实世界中的影响‖,并且应用了社会科学家的话语来表明应该采取行动,即:上段末句提到的问题,空前谈到存在问题,那么接下来应该解决问题,但是空后谈到这是一种耻辱,然后纠正应该解决问题,所以空格处应该承上启下,表达没有解决问题这个含义,因此C项―尽管存在这些因素,很多社会科学家不愿意解决此问题‖即为正确选项,该项中these factors指代上段末句提到的社会因素,而―很多科学家不愿意解决此问题‖就是下文提及的this。 43.【答案】B (However, the numbers are still small...) 【解析】本题可以通过原词复现和逻辑关系来确定答案。空前一句讲到― the number of papers including.....have increased rapidly ...‖,紧接着在选项B中也出现了―the number‖,属于原词复现。从逻辑关系的角度来看,选项B有一个明显的 转折词―however‖,这说明其表达的含义与空前信息相反,该选项提到―the numbers are still small‖(数量非常小),空前信息是―the number ... increased rapidly‖(数量增长非常迅速),两者在语意上构成了明显的转折关系。所以正确答 案为B。 44.【答案】D (During the late 1990s...) 【解析】本题设在段落中间,可以通过段落一致性代词指代来确定答案。该段 第一句提到:―The problem is not necessarily the amount of available funding‖,其表 达的含义是:事情的问题不是可用资金的数目。那接下来要讲的内容一定和资金 有关系。空后出现了代词―this is an adequate amount‖其含义是:这个资金数目是 足够的。那么this指代的内容一定和adequate amount相关,而选项D的题干是 ―national spending varied from 4% to 25%‖,其表述的内容正是资金的总量。因此 选项D为正确答案。 45.【答案】E (The idea is to force social to integrate...) 【解析】本题设空在段落中间,应瞻前顾后地依据连贯性原则确定答案。空前 的信息是指欧盟提议取消了之前设定的专门投资支持社会科学家的专栏项目,其 目的不是为了忽略社会科学家,而是完全相反,即文章中的―complete opposite‖, 根据语意衔接,接下来会说明欧盟这一做法的真正目的,并且这一目的对于社会 科学家一定是积极的。选项E中的the idea指代空前出现的―it was proposed that...‖,即欧盟的提案。除此之外,空后提到了collaborative endeavors,与选项 E中的短语integrate with构成了同义替换。空后信息中的global problems与选项 E中的health and demographic change, food security, ... and secure societies构成上 下义的关系,这也是解题的一道线索,因此,正确答案为选项E。 Section III Translation 46. yet, when one looks at the photographs of the gardens created by the homeless, it strikes one that, for all their diversity of styles, these gardens speak of various other fundamental urges, beyond that of decoration and creative expression. 【参考译文】然而,看着无家可归者绘制出的花园图片时,人们会突然想到, 尽管这些花园风格多样,它们都显示了人类除了装饰和创造性表达之外的其他各 种基本诉求 47. A sacred place of peace, however crude it may be, is a distinctly human need, as opposed to shelter, which is a distinctly animal need. 【参考译文】无论地方多么简陋不堪,寻求一片静谧圣土是人类特有的需求, 而动物需要的仅是仅是避难栖息之地。 48. The gardens of the homeless, which are in effect homeless gardens, introduce form into an urban environment where it either didn?t exist or was not discernible as such. 【参考译文】无家可归者的乐园,实际上是一个毫无家气息的地方,给城市环境 带来了一种新的形式。。 无家可归者描绘的花园实质上是无所依附的,这些花园把一种形式引入城市环 境中,而这样的城市环境中,形式要么根本不存在, 要么就完全不是以这种明显 的方式存在。 49. most of us give in to a demoralization of spirit which usually blame on some psychological conditions, until one day we find ourselves in a garden and feel the oppression vanish as if by magic. 【参考译文】我们大多数人会深陷于精神萎靡的状态,并常常将此归咎为一些 心理原因,直到某天我们发现自己置身花园中,感到如魔法般烦闷尽消 - 15 - 50. It is this implicit or explicit reference to nature that fully justifies the use of the word garden, though in a ―liberated‖ sense, to describe these synthetic constructions. 【参考译文】正是对自然的这种或隐晦含蓄或清晰直白的提及,充分证实了用 ―花园‖一词来描述这些虚拟建筑是合乎情理的,即使是从毫无拘泥的意义来讲 的。 Section IV Writing 51. 【参考范文】 Dear Professor Wang, I am writing on behalf of our college to invite you to be a judge for the English speech contest which will be held next week. We know that you are admired by all the students. As college students, we would like to improve our abilities in spoken English as well as written English. We would be grateful if you could be the judge for this contest to be held in Room 102, the TeachingBuildingon Saturday, June 4th, at 7p.m. We trust you will be disengaged and able to give us the pleasure of your company. We are looking forward to seeing you. Yours sincerely, Li Ming 52. 【参考范文】 Emerging from the cartoon is an eye-catching scene that a mass of graduates are at a critical turning point on the way to the future. A variety of choices, such as finding a job, going further education or abroad, and doing pioneering work, lie in front of them. The implication echoed by this cartoon can be summarized as a philosophic topic in our daily life:the success of a man is directly related to the choice made by himself. Nevertheless, I cherish a belief that we can not tell whether the selection is good or not, and as long as we adhere to our decision success will be realized step by step. Although making choice is essential to help determine the direction of our way, judged from the personal aspect, persistence functions as an indispensable driving force to keep up our spirit and to assist us to fulfill our study and work. However, some people, pacing up and down, are not industrious and try to find a short-cut success. In fact, only those who are hard-working and brave enough to encounter obstacles of all sorts are most likely to reach the summit of success. It occurs to many that the most fundamental thing to success is making a good choice. But I recommend that graduates as well as all the citizens should struggle to make their choice be a reality. 2012年全国硕士研究生入学统一考试英语试题 Section I Use of English Directions: Read the following text. Choose the best word(s) for each numbered blank and mark A, B, C or D on ANSWER SHEET 1. (10 points) The ethical judgments of the Supreme Court justices have become an important issue recently. The court - 16 - cannot _1_ its legitimacy as guardian of the rule of law _2_ justices behave like politicians. Yet, in several instances, justices acted in ways that _3_ the court?s reputation for being independent and impartial. Justice Antonin Scalia, for example, appeared at political events. That kind of activity makes it less likely that the court?s decisions will be _4_ as impartial judgments. Part of the problem is that the justices are not _5_by an ethics code. At the very least, the court should make itself _6_to the code of conduct that _7_to the rest of the federal judiciary. This and other similar cases _8_the question of whether there is still a _9_between the court and politics. The framers of the Constitution envisioned law _10_having authority apart from politics. They gave justices permanent positions _11_they would be free to _12_ those in power and have no need to _13_ political support. Our legal system was designed to set law apart from politics precisely because they are so closely _14_. Constitutional law is political because it results from choices rooted in fundamental social _15_ like liberty and property. When the court deals with social policy decisions, the law it _16_ is inescapably political-which is why decisions split along ideological lines are so easily _17_ as unjust. The justices must _18_ doubts about the court?s legitimacy by making themselves _19_ to the code of conduct. That would make rulings more likely to be seen as separate from politics and, _20_, convincing as law. 1. [A]emphasize 2. [A]when 3. [A]restored 4. [A]challenged 5. [A]advanced 6. [A]resistant 7. [A]resorts 8. [A]evade 9. [A]line 10. [A]by 11. [A]so 12. [A]serve 13. [A]confirm 14. [A]guarded 15. [A]concepts 16. [A]excludes 17. [A]dismissed 18. [A]suppress 19. [A]accessible 20.[A]byall means [B]maintain [B]lest [B]weakened [B]compromised [B]caught [B]subject [B]sticks [B]raise [B]barrier [B]as [B]since [B]satisfy [B]express [B]followed [B]theories [B]questions [B]released [B]exploit [B]amiable [B]at all costs [C]modify [C]before [C]established [C]suspected [C]bound [C]immune [C]loads [C]deny [C]similarity [C]though [C]provided [C]upset [C]cultivate [C]studied [C]divisions [C]shapes [C]ranked [C]address [C]agreeable [C]in a word [D] recognize [D] unless [D] eliminated [D] accepted [D]founded [D]prone [D]applies [D]settle [D]conflict [D]towards [D]though [D]replace [D]offer [D]tied [D]conceptions [D]controls [D]distorted [D]ignore [D]accountable [D]as a result Section II Reading Comprehension Part A Directions: Read the following four texts. Answer the questions below each text by choosing A, B, C or D. Mark your answers on ANSWER SHEET 1. (40 points) Text 1 - 17 - Come on –Everybody‘s doing it. That whispered message, half invitation and half forcing, is what most of us think of when we hear the words peer pressure. It usually leads to no good-drinking, drugs and casual sex. But in her new book Join the Club, Tina Rosenberg contends that peer pressure can also be a positive force through what she calls the social cure, in which organizations and officials use the power of group dynamics to help individuals improve their lives and possibly the word. Rosenberg, the recipient of a Pulitzer Prize, offers a host of example of the social cure in action: In South Carolina, a state-sponsored antismoking program called Rage Against the Haze sets out to make cigarettes uncool. In South Africa, an HIV-prevention initiative known as LoveLife recruits young people to promote safe sex among their peers. The idea seems promising,and Rosenberg is a perceptive observer. Her critique of the lameness of many pubic-health campaigns is spot-on: they fail to mobilize peer pressure for healthy habits, and they demonstrate a seriously flawed understanding of psychology.‖ Dare to be different, please don‘t smoke!‖ pleads one billboard campaign aimed at reducing smoking among teenagers-teenagers, who desire nothing more than fitting in. Rosenberg argues convincingly that public-health advocates ought to take a page from advertisers, so skilled at applying peer pressure. But on the general effectiveness of the social cure, Rosenberg is less persuasive. Join the Club is filled with too much irrelevant detail and not enough exploration of the social and biological factors that make peer pressure so powerful. The most glaring flaw of the social cure as it‘s presented here is that it doesn‘t work very well for very long. Rage Against the Haze failed once state funding was cut. Evidence that the LoveLife program produces lasting changes is limited and mixed. There‘s no doubt that our peer groups exert enormous influence on our behavior. An emerging body of research shows that positive health habits-as well as negative ones-spread through networks of friends via social communication. This is a subtle form of peer pressure: we unconsciously imitate the behavior we see every day. Far less certain, however, is how successfully experts and bureaucrats can select our peer groups and steer their activities in virtuous directions. It‘s like the teacher who breaks up the troublemakers in the back row by pairing them with better-behaved classmates. The tactic never really works. And that‘s the problem with a social cure engineered from the outside: in the real world, as in school, we insist on choosing our own friends. 21. According to the first paragraph, peer pressure often emerges as [A] a supplement to the social cure [B] a stimulus to group dynamics [C] an obstacle to school progress [D] a cause of undesirable behaviors 22. Rosenberg holds that public advocates should [A] recruit professional advertisers [B] learn from advertisers‘ experience [C] stay away from commercial advertisers [D] recognize the limitations of advertisements 23. In the author‘s view, Rosenberg‘s book fails to [A] adequately probe social and biological factors [B] effectively evade the flaws of the social cure [C] illustrate the functions of state funding [D]produce a long-lasting social effect - 18 - 24. Paragraph 5shows that our imitation of behaviors [A] is harmful to our networks of friends [B] will mislead behavioral studies [C] occurs without our realizing it [D] can produce negative health habits 25. The author suggests in the last paragraph that the effect of peer pressure is [A] harmful [B] desirable [C] profound [D] questionable Text 2 A deal is a deal-except, apparently ,when Entergy is involved. The company, a major energy supplier in New England, provoked justified outrage in Vermont last week when it announced it was reneging on a longstanding commitment to abide by the strict nuclear regulations. Instead, the company has done precisely what it had long promised it would not challenge the constitutionality of Vermont‘s rules in the federal court, as part of a desperate effort to keep its Vermont Yankee nuclear power plant running. It‘s a stunning move. The conflict has been surfacing since 2002, when the corporation bought Vermont‘s only nuclear power plant, an aging reactor in Vernon. As a condition of receiving state approval for the sale, the company agreed to seek permission from state regulators to operate past 2012. In 2006, the state went a step further, requiring that any extension of the plant‘s license be subject to Vermont legislature‘s approval. Then, too, the company went along. Either Entergy never really intended to live by those commitments, or it simply didn‘t foresee what would happen next. A string of accidents, including the partial collapse of a cooling tower in 207 and the discovery of an underground pipe system leakage, raised serious questions about both Vermont Yankee‘s safety and Entergy‘s management– especially after the company made misleading statements about the pipe. Enraged by Entergy‘s behavior, the Vermont Senate voted 26 to 4 last year against allowing an extension. Now the company is suddenly claiming that the 2002 agreement is invalid because of the 2006 legislation, and that only the federal government has regulatory power over nuclear issues. The legal issues in the case are obscure: whereas the Supreme Court has ruled that states do have some regulatory authority over nuclear power, legal scholars say that Vermont case will offer a precedent-setting test of how far those powers extend. Certainly, there are valid concerns about the patchwork regulations that could result if every state sets its own rules. But had Entergy kept its word, that debate would be beside the point. The company seems to have concluded that its reputation in Vermont is already so damaged that it has noting left to lose by going to war with the state. But there should be consequences. Permission to run a nuclear plant is a poblic trust. Entergy runs 11 other reactors in the United States, including Pilgrim Nuclear station in Plymouth. Pledging to run Pilgrim safely, the company has applied for federal permission to keep it open for another 20 years. But as the Nuclear Regulatory Commission (NRC) reviews the company‘s application, it should keep it mind what promises from Entergy are worth. 26. The phrase ,reneging on‖(Line 3.para.1) is closest in meaning to - 19 - [A] condemning. [B] reaffirming. [C] dishonoring. [D] securing. 27. By entering into the 2002 agreement, Entergy intended to [A] obtain protection from Vermont regulators. [B] seek favor from the federal legislature. [C] acquire an extension of its business license . [D] get permission to purchase a power plant. 28. According to Paragraph 4, Entergy seems to have problems with its [A] managerial practices. [B] technical innovativeness. [C] financial goals. [D] business vision 29. In the author‘s view, the Vermont case will test [A] Entergy‘s capacity to fulfill all its promises. [B] the mature of states‘ patchwork regulations. [C] the federal authority over nuclear issues . [D] the limits of states‘ power over nuclear issues. 30. It can be inferred from the last paragraph that [A] Entergy‘s business elsewhere might be affected. [B] the authority of the NRC will be defied. [C] Entergy will withdraw its Plymouth application. [D] Vermont‘s reputation might be damaged. Text 3 In the idealized version of how science is done, facts about the world are waiting to be observed and collected by objective researchers who use the scientific method to carry out their work. But in the everyday practice of science, discovery frequently follows an ambiguous and complicated route. We aim to be objective, but we cannot escape the context of our unique life experience. Prior knowledge and interest influence what we experience, what we think our experiences mean, and the subsequent actions we take. Opportunities for misinterpretation, error, and self-deception abound. Consequently, discovery claims should be thought of as protoscience. Similar to newly staked mining claims, they are full of potential. But it takes collective scrutiny and acceptance to transform a discovery claim into a mature discovery. This is the credibility process, through which the individual researcher‘s me, here, now becomes the community‘s anyone, anywhere, anytime. Objective knowledge is the goal, not the starting point. Once a discovery claim becomes public, the discoverer receives intellectual credit. But, unlike with mining claims, the community takes control of what happens next. Within the complex social structure of the scientific community, researchers make discoveries; editors and reviewers act as gatekeepers by controlling the publication process; other scientists use the new finding to suit their own purposes; and finally, the public (including other scientists) receives the new discovery and possibly accompanying technology. As a discovery claim works it through the community, the interaction and confrontation between shared and competing beliefs about the science and the technology involved transforms an individual‘s discovery claim into the community‘s credible discovery. Two paradoxes exist throughout this credibility process. First, scientific work tends to focus on some aspect of prevailing Knowledge that is viewed as incomplete or incorrect. Little reward accompanies duplication and confirmation of what is already known and believed. The goal is new-search, not re-search. Not surprisingly, newly published discovery claims and credible discoveries that appear to be important and convincing will always be open to challenge and potential modification or refutation by future researchers. Second, novelty itself frequently provokes disbelief. Nobel Laureate and physiologist Albert - 20 - Azent-Gyorgyi once described discovery as ,seeing what everybody has seen and thinking what nobody has thought.‖ But thinking what nobody else has thought and telling others what they have missed may not change their views. Sometimes years are required for truly novel discovery claims to be accepted and appreciated. In the end, credibility ,happens‖ to a discovery claim – a process that corresponds to what philosopher Annette Baier has described as the commons of the mind. ,We reason together, challenge, revise, and complete each other‘s reasoning and each other‘s conceptions of reason.‖ 31. According to the first paragraph, the process of discovery is characterized by its [A] uncertainty and complexity. [B] misconception and deceptiveness. [C] logicality and objectivity. [D] systematicness and regularity. 32. It can be inferred from Paragraph 2 that credibility process requires [A] strict inspection. [B]shared efforts. [C] individual wisdom. [D]persistent innovation. 33.Paragraph 3 shows that a discovery claim becomes credible after it [A] has attracted the attention of the general public. [B]has been examined by the scientific community. [C] has received recognition from editors and reviewers. [D]has been frequently quoted by peer scientists. 34. Albert Szent-Györgyi would most likely agree that [A] scientific claims will survive challenges. [B]discoveries today inspire future research. [C] efforts to make discoveries are justified. [D]scientific work calls for a critical mind. 35.Which of the following would be the best title of the test? [A] Novelty as an Engine of Scientific Development. [B]Collective Scrutiny in Scientific Discovery. [C] Evolution of Credibility in Doing Science. [D]Challenge to Credibility at the Gate to Science. Text 4 If the trade unionist Jimmy Hoffa were alive today, he would probably represent civil servant. When Hoffa‘s Teamsters were in their prime in 1960, only one in ten American government workers belonged to a union; now 36% do. In 2009 the number of unionists in America‘s public sector passed that of their fellow members in the private sector. In Britain, more than half of public-sector workers but only about 15% of private-sector ones are unionized. There are three reasons for the public-sector unions‘ thriving. First, they can shut things down without suffering much in the way of consequences. Second, they are mostly bright and well-educated. A quarter of America‘s public-sector workers have a university degree. Third, they now dominate left-of-centre politics. Some of their ties go back a long way. Britain‘s Labor Party, as its name implies, has long been associated with trade unionism. Its current leader, Ed Miliband, owes his position to votes from public-sector unions. At the state level their influence can be even more fearsome. Mark Baldassare of the Public Policy Institute of California points out that much of the state‘s budget is patrolled by unions. The teachers‘ unions keep an eye on schools, the CCPOA on prisons and a variety of labor groups on health care. - 21 - In many rich countries average wages in the state sector are higher than in the private one. But the real gains come in benefits and work practices. Politicians have repeatedly ,backloaded‖ public-sector pay deals, keeping the pay increases modest but adding to holidays and especially pensions that are already generous. Reform has been vigorously opposed, perhaps most egregiously in education, where charter schools, academies and merit pay all faced drawn-out battles. Even though there is plenty of evidence that the quality of the teachers is the most important variable, teachers‘ unions have fought against getting rid of bad ones and promoting good ones. As the cost to everyone else has become clearer, politicians have begun to clamp down. In Wisconsin the unions have rallied thousands of supporters against Scott Walker, the hardline Republican governor. But many within the public sector suffer under the current system, too. John Donahue at Harvard‘s Kennedy School points out that the norms of culture in Western civil services suit those who want to stay put but is bad for high achievers. The only American public-sector workers who earn well above $250,000 a year are university sports coaches and the president of the United States. Bankers‘ fat pay packets have attracted much criticism, but a public-sector system that does not reward high achievers may be a much bigger problem for America. 36. It can be learned from the first paragraph that [A] Teamsters still have a large body of members. [B] Jimmy Hoffa used to work as a civil servant. [C] unions have enlarged their public-sector membership. [D]the government has improved its relationship with unionists. 37. Which of the following is true of Paragraph 2? [A] Public-sector unions are prudent in taking actions. [B] Education is required for public-sector union membership. [C] Labor Party has long been fighting against public-sector unions. [D]Public-sector unions seldom get in trouble for their actions. 38. It can be learned from Paragraph 4 that the income in the state sector is [A] illegally secured. [B] indirectly augmented. [C] excessively increased. [D]fairly adjusted. 39. The example of the unions in Wisconsin shows that unions [A]often run against the current political system. [B]can change people‘s political attitudes. [C]may be a barrier to public-sector reforms. [D]are dominant in the government. 40. John Donahue‘s attitude towards the public-sector system is one of [A]disapproval. [B]appreciation. [C]tolerance. [D]indifference. Part B Directions: In the following text, some sentences have been removed. For Questions 41-45, choose the most suitable one from the list A-G to fit into each of the numbered blanks. There are two extra choices, which do not fit in any of the blanks. Mark your answers on ANSWER SHEET1.(10 points) Think of those fleeting moments when you look out of an aeroplane window and realise that you are - 22 - flying, higher than a bird. Now think of your laptop, thinner than a brown-paper envelope, or your cellphone in the palm of your hand. Take a moment or two to wonder at those marvels. You are the lucky inheritor of a dream come true. The second half of the 20th century saw a collection of geniuses, warriors, entrepreneurs and visionaries labour to create a fabulous machine that could function as a typewriter and printing press, studio and theatre, paintbrush and gallery, piano and radio, the mail as well as the mail carrier. (41) The networked computer is an amazing device, the first media machine that serves as the mode of production, means of distribution, site of reception, and place of praise and critique. The computer is the 21st century‘s culture machine. But for all the reasons there are to celebrate the computer, we must also tread with caution. (42)I call it a secret war for two reasons. First, most people do not realise that there are strong commercial agendas at work to keep them in passive consumption mode. Second, the majority of people who use networked computers to upload are not even aware of the significance of what they are doing. All animals download, but only a few upload. Beavers build dams and birds make nests. Yet for the most part, the animal kingdom moves through the world downloading. Humans are unique in their capacity to not only make tools but then turn around and use them to create superfluous material goods - paintings, sculpture and architecture - and superfluous experiences - music, literature, religion and philosophy. (43) For all the possibilities of our new culture machines, most people are still stuck in download mode. Even after the advent of widespread social media, a pyramid of production remains, with a small number of people uploading material, a slightly larger group commenting on or modifying that content, and a huge percentage remaining content to just consume. (44) Television is a one-way tap flowing into our homes. The hardest task that television asks of anyone is to turn the power off after he has turned it on. (45) What counts as meaningful uploading? My definition revolves around the concept of "stickiness" - creations and experiences to which others adhere. [A] Of course, it is precisely these superfluous things that define human culture and ultimately what it is to be human. Downloading and consuming culture requires great skills, but failing to move beyond downloading is to strip oneself of a defining constituent of humanity. [B] Applications like tumblr.com, which allow users to combine pictures, words and other media in creative ways and then share them, have the potential to add stickiness by amusing, entertaining and enlightening others. [C] Not only did they develop such a device but by the turn of the millennium they had also managed to embed it in a worldwide system accessed by billions of people every day. [D] This is because the networked computer has sparked a secret war between downloading and uploading - between passive consumption and active creation - whose outcome will shape our collective future in ways we can only begin to imagine. - 23 - [E] The challenge the computer mounts to television thus bears little similarity to one format being replaced by another in the manner of record players being replaced by CD players. [F] One reason for the persistence of this pyramid of production is that for the past half-century, much of the world‘s media culture has been defined by a single medium - television - and television is defined by downloading. [G]The networked computer offers the first chance in 50 years to reverse the flow, to encourage thoughtful downloading and, even more importantly, meaningful uploading. Part C Directions: Read the following text carefully and then translate the underlined segments into Chinese. Your translation should be written clearly on ANSWER SHEET 2. (10 points) Since the days of Aristotle, a search for universal principles has characterized the scientific enterprise. In some ways, this quest for commonalities defines science. Newton‘s laws of motion and Darwinian evolution each bind a host of different phenomena into a single explicatory frame work. (46)a theory would be a simplification, given the dimensions and universes that it might entail, nonetheless, unification of sorts remains a major goal. This tendency in the natural sciences has long been evident in the social sciences too. (47) Just as the bewildering variety of human courtship rituals might all be considered forms of sexual selection, perhaps the world‘s languages, music, social and religious customs and even history are governed by universal features. (48) That, at least, is the hope. But a comparative study of linguistic traits published online today supplies a reality check. Russell Gray at the University of Auckland and his colleagues consider the evolution of grammars in the light of two previous attempts to find universality in language. The most famous of these efforts was initiated by Noam Chomsky, who suggested that humans are born with an innate language—acquisition capacity that dictates a universal grammar. A few generative rules are then sufficient to unfold the entire fundamental structure of a language, which is why children can learn it so quickly. (49) Gray and his colleagues have put them to the test by examining four family trees that between them represent more than 2,000 languages.(50)of these patterns is borne out by the analysis, suggesting that the structures of the languages are lire age-specific and not governed by universals - 24 - (48) (49) 2012考研英语真题答案 Section ? Use of English 2012年的完型填空是有关美国司法官伦理和政治关系的一篇文章,出自New York Times, June, 30th , 2011的―Ethics, Politics and the Law‖一文。选材回归了2000年完型曾出过的法律类文章,而且和当年一样,也是包含几个小段落,不像以往的文章,三段或者四段论,脉络比较清晰,结构容易把握。而且,较去年比较―平易近人‖的文章,这篇法律类文章背后有一定的背景知识,比较关注时事或者对这一块儿有所了解的同学,会相应得心应手一些。另外,20道题目中,多达13题都是在考查动词,虽然选项中基本不存在干扰项,除了15题一道考查两词的辨析之外,其他的选项含义都差别甚远,按理说值得高兴。但是这些考查动词的题目中,许多都考查对于熟词僻义的掌握情况,往年就是08年出现了3处,今年也出现3处。仅有2道题考查逻辑词,而且这两道题是 - 25 - 送分题,不需要考虑太多。一向是命题人偏爱的以‖able‖作后缀的形容词依然出现(19题)。下面就真题作一个详细解析。 和以往一样,第一句话不设空,帮助同学们理解全文探讨的话题: 美国高等法庭司法官的伦理道德问题。 题1选B。maintain. 此空有赖于对后文的理解。这直接体现了我们作完型的整体思路,也就是首先通读全文。尤其是看到最后一段直接给出提议:希望法官和政治划清界限从而保证自己的权威性,因此全文的导向和逻辑就非常清晰了。同时,题2答案(when)也顺势而出:如果法官们和政治家一样,法庭就不能捍卫自己作为法律卫道士的权威。 题2选A。这里的when其实表示条件关系,即―如果……。‖ 题3选择 weakened。上下文语义题+词义辨析。选项含义差别较大,要求对上 下文逻辑关系掌握清楚。Yet表示一个转折:―即使这样,还是有很多法官这样做,损害了法庭独立和公正的名声。‖ 本题如果能把导向把握准,即可定位在B和D两项,D项eliminated 过于绝对,排除。 题4 选D. accepted。 上下文语义。依然是通过上下文逻辑关系,按段精度解题。这段用两个法官参与政治事件的例子说明他们的行为使得法院的裁决不偏不倚而被认可变得不太可能。A选项challenged 和C项suspected意义相反,和主旨矛盾,即肯定了法官参与政治行为。B项compromised ―调停‖ 和主题无关。 题5 选C。上下文语义题。本段属于总分形式。首先提出法官参与政治导致的问题,然后分析问题的原因:―这种情况有一部分是由于法官们不受伦理道德准则的约束。‖ 其他三项不合逻辑,均不具干扰性。 题 6 选B。完型填空在历史上对于subject这个词一向情有独钟。04年,05年,08年以及10年选项中都出现,只有05年落选。而且这3年考查的意思都一致,即―易遭受……‖,考查形式有―is subjected to‖, ―is subject to‖ 以及 ―subject sb. to sth。(使得某人容易遭受)‖。今年依然是―subject‖入选,但是这次取了大家最熟悉的意思,即―使服从,受…管制‖,―法院应该依行为准则办事‖。至于选项A resistant 和 C immune两个选项和subject 这三个词在04年的第8道题也同时出现。―Be immune to‖指―对…免疫,不起作用‖。可见很多时候命题人不总以 题14 选 D. tied。上下文语义题。―法制系统旨在将政治与法律相分隔,因为它们彼此联系太紧。‖其他三项完全构不成干扰。 题15 选 A. concepts。词义辨析题。―liberty‖和―property‖同属社会基本理念,而不是 theories (理论) 或divisions (分支)。D项conception 常指具体的或个人的理念而非笼统意义的理念。 题16 选 C. shapes。考查熟词僻义。―Shape a plan‖, 作出计划。本词所在的句子为省略了that 的定语从句。―当法院处理社会政策决议的问题时,它们制定出来的法律将不可避免的政治化。‖ 题17 选 A. dismissed。 考查熟词僻义。Dismiss 在这里意为―拒绝考虑,遭拒‖。―这就解释了为什么不同意识形态导致的决议分歧总是被认为不公正因而不予考虑。 题18 选 C. address。 依然考查熟词僻义。最后一段提出建议。希望法官们必须处理人们对于法庭合法性的质疑。―address‖ 一词作为―处理,解决‖的意思在完型中不是第一次出现。06年的完型中最后一段―address the - 26 - needs of the homeless‖, 解决无家可归者的需求。只不过当年这里并没有设空。其他三项均不符合逻辑。 题19. 选D. accountable. 考查点:同义复现+熟词僻义。此题和第二段的题6构成同义复现。―be accountable to…‖不是―可解释的‖,而是指―有义务,对…负责‖。―be agreeable to‖ 是―愉快的;同意的‖;虽然也有―适合的,符合品位的‖的意思,但题中to后面接的是code,不合逻辑。 ―agreeable to everybody: 适合于所有人‖。A项 ―be accessible to…,可接近…,能得到的‖ 和B项―amiable‖ 和蔼可亲的,与主题无关。 题20. 选D. as a result。考查逻辑关系词。A项 by all means 和B 项at all costs 用法一致,切含义相近,首先排除。C项in a word 是―总之‖,总结全文下结论 时用,而且多用在句首。这里and连接词前面的 - 27 - 个词的考生就不敢选择A答案了。 24. Paragraph 5 shows that our imitation of behaviors 第五段表明对于行为的模仿: [A] is harmful to our networks of friends 对于朋友的网络是有害的 [B] will mislead behavioral studies 会误导对于行为的研究 [C] occurs without our realizing it 在没有意识到的时候就出现了 [D] can produce negative health habits 会产生不良的健康习惯 解析:答案为C。细节题;通过题干中imitation和behaviors可定位于第五段最后一句。This is a subtle form of peer pressure: we unconsciously imitate the behavior we see every day。对 unconsciously,subtle这个词在我们04年第二篇文章的第二题的答案中出现过,04年第二篇文章关于社会中按字母排序现象的这篇文章时应该有印象,我们的27题的答案选择的是D答案:Some form of discrimination is to subtle to recognize,大家可以看一下这道题和以前考过的题出现的相似性。 25. The author suggests in the last paragraph that the effect of peer pressure is 作者在最后一段中认为同龄人压力的效果是: [A] harmful 有害的[B] desirable 令人满意的 [C] profound 意义深远的 [D] questionable 值得怀疑的 解析:答案为D。态度题 考查作者对同龄人压力效果的态度,对应Far less certain, however, is…,第一句中in virtuous(正直的,有效的)directions。 但是大家不认识也不会影响句子的理解,因为紧接着第二句在举例, 就像那些老师为了将后排捣乱的学生分开而将他们和他们与表现好的学生安排在一块, The tactic never really works…(这些策略(方法)不是真正起作用的)表明作者认为同龄人压力效果是不确定的。 Text 2 2012年考研英语阅读Text2的文章来源于Boston.com上的一篇评论性文章,题为Vermont Yankee plant?s owner must honor its own promises. 文章的题材属于信息科技类文章。讲述的是有关New England 一家公司购买了核电站后,不遵守核能源的一些相关规定后所引起的一些争议。就文章本身而言,放置在Reading的第二篇,难度不能算大。因为行文中,专业性术语和词汇不多,基本不影响考生对文章总体的理解。但是本篇文章中出现了一些相对比较生僻的词汇,如:renege on, aging reaction 等,但是通过上下文的揣摩和推测,要判断出这些生词的意思应该比较容易。所以整体而言,Text2的难度系数中等。题材合理。依旧是开篇直接引出问题所在。所以第一段应该是大家应该多加留心和关注的焦点。也有助于大家理解全文,把握文章的中心大意。 26. C 语义题。考的是一个词汇在文章中的特定含义。Reneging on 出现在文章中的第一段。我们 先从选项入手。 A. condemning 谴责 B. reaffirming 重申,再肯定 C. dishonoring 拒付,丢脸,不光彩 D. securing 保卫,保护 从文中我们可以看到,公司的这种行为激起了公愤,是在当它…一个长期以来坚持遵守的承诺,有关遵守严格的核能源惯例。 根据常识可以判断:此处一定是一个贬义词,所以才导致引起了群众的愤怒,并且需要和commitment构成动宾关系,所以综合考虑,可以先排除A和D选项。而B选项―重申‖,是一个中性词,所以本句话的意思是:这个公司违反了惯例。正确选项为:C 27. A 事实细节题。―加入2002年的协议,Entergy本打算,,‖需要把信息定位到文章第3段,第四行,―…the company agreed to seek permission from state regulators to operate past 2012.‖公司同意2012年以后的运营需要在征得州相关管理规定的允许为前提条件。可以判断出:正确选项为A。即:打算得到来自Vermont 的管理规定的保护。应该是与原文属于同义替换。B选项错在 ―Federal‖一词上,扩大了范围。C选项的意思是:延长商业执照的有效期,误解了原文意思。D选项提到获得购买一个电厂的许可与原文中―已经购买了唯一的一个核电站‖的描述不符,所以排除。 28. A 推断性实施细节题。文章第四段在讲:公司出现的另外一些如管道泄露等等问题也给公司造成了一定的影响。所以可以判断出,不仅仅是它违背核能源惯例这一项使得它进入了当下的困境中。而是公司本身 - 28 - 和财务,前景以及技术方面都没有直接的关联。 29.B 事实细节题。作者认为Vermont 案例能检验出,,, 我们可以把信息定位到文章第5段第4-5行:‖…legal scholars say that Vermont case will offer a precedent-setting test of how far these powers extend…‖ 我们可以得到是一种对权力的 检测 工程第三方检测合同工程防雷检测合同植筋拉拔检测方案传感器技术课后答案检测机构通用要求培训 ,检测各个州所拥有的这种权利到底能延伸多远。所以对象应该是针对州,而不是公司。首先排除A 和C. 余下的B选项和D选项当中,B的意思是各州这些比较杂乱的制度的成熟程度,而D选项指的是在核问题上,各州权力的局限性。结合文章,我们最终选择B选项,因为文章更多的是在指各州如何来运用自己的权利,而非强调局限性。 30. B 推断题。提到一个关键词:NRC 需要定位到文章最后一个段落。文章最后一句提到:But as the NRC reviews the company?s application, it should keep it mind what from Entergy are worth。‖ 用排除法,可以先排除选项D 名誉可能会损坏与原文不符,因为原文说道 is already damaged. 接下来,从文章中可以看到公司提出的申请是有关年限延长的申请而非能不能在Plymouth 的申请,所以可以排除选项C. 而A选项中提到公司在其他地方的生意会受到影响恰恰与文章描述相反。所以,综上所述,正确 选项为B 也就是说这一系列的事件已经对NRC 的权威构成了一种挑战。 Text 3 文章解析: 第四段描述信度演变过程存在的两个悖论。首先,科研工作者致力于研究不完善或错误的领域,他们极少对已经验证过的东西有兴趣。因此,新的研究成果会 成为他们的聚焦目标,接受来自未来科研工作的不断挑战或否定。再者,新的研究成果本身就会引起质疑,你的研究成果即使是对的,也不一定能说服他人。有时候,一项正确的新的研究成果需要经过几年,几十年才能为大家所接受。最后,这项研究成果终于具备了信度。 31. A,答案在文章第一段第四行,ambiguous and complicated 与 选项A. uncertainty and complexity同义词转换,为正确选项。 32. B 第二段第三句 it takes collective scrutiny and acceptance to …选项B表达了相同意思,为同义句。再者,第二段为承上启下段,后面的第三段对这个观点进行了详细论述。 33. B,此为段落概括的考察,A是无关选项,文中没有提到;选项C,D为干扰项,只是文中所说的一个方面,以小概念偷换大概念,此为考研英语干扰项常见模式。 34. D,此题较难,很容易选错。首先,选项A,B可以排除,因为A,B是第一个悖论的观点,Albert的话属于第二个悖论。英语文章具有很强的逻辑性,且非常清晰,标点符号是最好的识别标志。另外,根据句子意思,选项C与段落关联不大,因此D为最佳答案。 35. C, 此题为全文主旨题,较为容易。 Text 4 这是出自经济学人杂志2011年3月11号的一篇文章,题为: Enemies of progress :The biggest barrier to public- sector reform are the unions 36. It can be learned from the first paragraph that 从第一段能推出什么 [A] Teamsters still have a large body of members. Teamsters依然有大量的会员 [B] Jimmy Hoffa used to work as a civil servant. Jimmy Hoffa曾经是一名公务员 [C] unions have enlarged their public-sector membership. 工会已经增加了其公共行业的会员 [D]the government has improved its relationship with unionists。 - 29 - 政府已经提升了其同工会会员之间的关系。 解析:段落推理题 优先对应主题和转折;但是都没有,只能逐个排除: [A] Teamsters still have a large body of members. Teamsters依然有大量的会员 文中没有提及 [B] Jimmy Hoffa used to work as a civil servant. Jimmy Hoffa曾经是一名公务员 he would probably represent civil servant他可能会代表公务员,并不意味着他是公务员。 [C] unions have enlarged their public-sector membership。工会已经增加了其公共行业的会员 only one in ten American government workers belonged to a union; now 36% do。 现在有36%确实增加了。正解! [D]the government has improved its relationship with unionists。 政府已经提升了其同工会会员之间的关系。 没有提及政府和工会会员之间的关系排除。 37. Which of the following is true of Paragraph 2?根据第二段,下面那个答案是正 确的 [A] Public-sector unions are prudent in taking actions。 公共行业的工会在采取行动是会更加谨慎 [B] Education is required for public-sector union membership。 公共行业的工会会员需要教育 [C] Labor Party has long been fighting against public-sector unions。 工党同公共行业的工会长期斗争 [D]Public-sector unions seldom get in trouble for their actions。 公共行业的工会很少由于他们的行为而惹麻烦。 解析:细节题中的which 题,只能一个个排除: 先看[B] Education is required for public-sector union membership。 公共行业的工会会员需要教育 文中说到了Second, they are mostly bright and well-educated. A quarter of America?s public-sector workers have a university degree。但是并没有说需要教育。 再看[C] Labor Party has long been fighting against public-sector unions。 工党同公共行业的工会长期斗争 Britain?s Labor Party, as its name implies, has long been associated with trade unionism。 选项和原文间的表述是相反的。 再看[D]Public-sector unions seldom get in trouble for their actions。 公共行业的工会很少由于他们的行为而惹麻烦。 First, they can shut things down without suffering much in the way of consequences。 首先他们可以罢工而不遭受后果。 同义替换成功。 最后看[A] Public-sector unions are prudent in taking actions。 公共行业的工会在采取行动是会更加谨慎 这是一个很强烈的干扰项,但是文中并没有提到工会很谨慎,原文----选项A----选项B 最后答案只能选A而不能选B;B 为未提及选项。 38. It can be learned from Paragraph 4 that the income in the state sector is 从第四段可以推知公共行业的收入 [A] illegally secured。非法的安全 [B] indirectly augmented。间接的增加了 [C] excessively increased。过度的增加了 [D]fairly adjusted。公平的调整了 解析:细节题 对应keeping the pay increases modest but adding to holidays and especially pensions that are - 30 - already generous。保持收入小幅上涨,但是增加假期特别是已经很慷慨的补助了。 主要是一个单词大家不认识augmented出题人开始耍无赖了,考研词汇大纲中 并无此单词,不过根据另外一个副词是可以推出答案的。 39. The example of the unions in Wisconsin shows that unions 威斯康辛州的工会的例子表明工会 [A]often run against the current political system。 通常和现行的政治系统相违背 [B]can change people?s political attitudes。 能够改变人们的政治态度 [C]may be a barrier to public-sector reforms。 可能是公共领域改革的障碍 [D]are dominant in the government。 受到政府的主导 解析:例证题 找观点 Reform has been vigorously opposed;之后的例子就应该围绕改革来讲。 40. John Donahue?s attitude towards the public-sector system is one of John Donahue对于公共行业系统的他的态度是: [A]disapproval。不赞同 [B]appreciation。欣赏 [C]tolerance。容忍 [D]indifference。漠不关心 解析:局部态度题,在转折的地方but a public-sector system that does not reward high achievers may be a much bigger problem for America出现bigger problem 表明了态度为disapproval。. Part B 41. C 复现结构定位法+代词指代定位法 41题出现在段尾。复现结构―The second half of the 20th century‖20世纪后半期,与―the turn of the millennium‖千年之交。代词 ―such a device‖指代―fabulous machine、typewriter and printing press‖等设备。因此,正确答案为C。 42. D 复现结构定位法 42题出现在段中,根据就近原则,线索为― a secret war‖一场秘密的战争。正确答案为D。 43. A 逻辑关系定位法+复现结构定位法+代词指代定位法 43题出现在文章段尾,前一句中提到superfluous material goods多余的材料,选项A中宾语these superfluous things这些多余的东西,可以在43题前一句中找到所指。前后两句话形成一个因果的逻辑关系,用连接词Of course ,因此正确答案为A。 44. F 代词指代定位法+复现结构定位法 44题出现在段尾,起到承上启下的作用。第一个复现点在44题前一句主语a pyramid of production 在选项F中宾语this pyramid of production复现。第二个复现点在television 是下一段落的主语,所以F选项为正确答案。 45. G 复现结构定位法 45题出现在段尾,起承上启下的作用。前文提到电视是one-way tap flowing单向流动,而互联网可以reverse the flow 逆转这个流动。the flow的所指one-way tap flowing单向流动。第二个复现结构在于启下,45题下一段为特殊疑问句,中心词为meaningful uploading有意义的上传,选项G提及meaningful uploading这一信息,所以,选项G为正确答案。 Part C 46. 【解析】本题语法结构并不复杂,关键在于对于概念陌生的unification的正确把握。unification是动词unify的名词形式,意为―统一、一元化‖。然而,结合上段最后一句:Newton‘s laws of motion and Darwinian evolution each bind a host of different phenomena into a single explicatory framework。以及本句并列结构后半部分,可以把握在 【参考译文】在物理学上,有一种方法把这种追求一致解释理论的愿望发挥到了极致,它试图找到一种万物的理论——用唯一的生成等式解释我们能看到的所有事物。 47.【解析】本句语法结构并不复杂,for为连词,意为―因为‖。后半句涉及到对于形式主语的考查。 【参考译文】在此,达尔文主义似乎提供了一种合理解释,因为如果整个人类都有着相同的起源,那么认为,文化多样性也可能追溯到人类更有局限性的开端,似乎是有理由的。 48.【解析】本句主语为不定式短语to filter out what is unique from what is shared,谓语部分为enable sb to do sth。其中考查了4个宾语从句,语法结构方面较上两句复杂。句中代词it指代complex cultural behavior;term一词,考生可联想短语in terms of(就…而言,在…方面)来选择恰当词义;filter out意为:―过滤掉、筛除、淘汰‖。 【参考译文】如果我们从共性之中去除个性,那么或许我们能够理解复杂的文化行为如何产生,以及何种因素在进化或认识的角度,引导了这种复杂的文化行为。 49.【解析】本题考点涉及代词的还原,现在分词做状语,过去分词做后置定语,非限制性定语从句以及被动。句首the second,回上文寻找,可在上段第一句找到The most famous of these efforts was initiated by Noam Chomsky,其中initiate为―发起、创始‖,可推知此处为the second effort,即上文的two previous attempts;shared by many language修饰traits;which引导的定语从句,同样修饰traits;定语从句的中的被动语态可采用主动的译法,添加主语―人们‖。 【参考译文】做出第二次努力的是乔舒亚?格林伯雷。他采用更加经验的方法来研究这种普遍性,他发现了多种语言的共同特征(特别是词序上的特征),而人们认为这种共同特征体现了认识由于认知局限所产生的偏见。 50.【解析】语法结构方面,本句考查了定语从句,过去分词做后置定语,并列结构等。词汇方面,family tree词义的选择,可借助上文by examining four family trees that between them represent more than 2,000 languages,可判断为语言的谱系,后文代词it指代family tree。 【参考译文】乔姆斯基的语法应该表明语言变化的模式,这种模式独立于语言谱系,或者独立于由该谱系所产生的路径,而格林伯格的普遍性理论则认为在特定种类的词序关系之间,有着紧密的相互依赖关系。 2011年全国硕士研究生入学统一考试英语(一)试题 Section I Use of English Directions: Read the following text. Choose the best word(s) for each numbered blank and mark [A], [B], [C] or [D] on ANSWER SHEET 1. (10 points) Ancient Greek philosopher Aristotle viewed laughter as ,a bodily exercise precious to health.‖ But __6__, instead of straining muscles to build them, as exercise does, laughter apparently accomplishes the __7__, studies dating back to the 1930‘s indicate that laughter__8___ muscles, decreasing muscle tone for up to 45 minutes after - 32 - the laugh dies down. Such bodily reaction might conceivably help _9__the effects of psychological stress. Anyway, the act of laughing probably does produce other types of ___10___ feedback, that improve an individual‘s emotional state. __11____one 19th century that humans do not cry ___13___they are sad but they become sad when the tears begin to flow. Although sadness also ____14___ tears, evidence suggests that emotions can flow __15___ muscular responses. In an experiment published in 1988,social psychologist Fritz Strack of the University of würzburg in Germany asked volunteers to __16___ a pen either with their teeth-thereby creating an artificial smile – or with their lips, which would produce a(n) __17___ expression. Those forced to exercise their enthusiastically to funny catoons than did those whose months were contracted in a frown, ____19___ that expressions may influence emotions rather than just the other way around __20__ , the physical act of laughter could improve mood. 1([A]among [B]except [C]despite [D]like 2([A]reflect [B]demand [C]indicate [D]produce 3([A]stabilizing [B]boosting [C]impairing [D]determining 4([A]transmit [B]sustain [C]evaluate [D]observe 5([A]measurable [B]manageable [C]affordable [D]renewable 6([A]In turn [B]In fact [C]In addition [D]In brief 7([A]opposite [B]impossible [C]average [D]expected 8([A]hardens [B]weakens [C]tightens [D]relaxes 9([A]aggravate [B]generate [C]moderate [D]enhance 10([A]physical [B]mental [C]subconscious [D]internal 11([A]Except for [B]According to [C]Due to [D]As for 12([A]with [B]on [C]in [D]at 13([A]unless [B]until [C]if [D]because 14([A]exhausts [B]follows [C]precedes [D]suppresses 15([A]into [B]from [C]towards [D]beyond 16([A]fetch [B]bite [C]pick [D]hold 17([A]disappointed [B]excited [C]joyful [D]indifferent 18([A]adapted [B]catered [C]turned [D]reacted 19([A]suggesting [B]requiring [C]mentioning [D]supposing 20([A]Eventually [B]Consequently [C]Similarly [D]Conversely Section II Reading Comprehension Part A Directions: Read the following four texts. Answer the questions below each text by choosing [A], [B], [C] or [D]. Mark your answers on ANSWER SHEET 1. (40 points) Text 1 The decision of the New York Philharmonic to hire Alan Gilbert as its next music director has been the talk of the classical-music world ever since the sudden announcement of his appointment in 2009. For the most part, the response has been favorable, to say the least. ,Hooray! At last!‖ wrote Anthony Tommasini, a sober-sided classical-music critic. One of the reasons why the appointment came as such a surprise, however, is that Gilbert is comparatively little known. Even Tommasini, who had advocated Gilbert‘s appointment in the Times, calls him ,an unpretentious musician with no air of the formidable conductor about him.‖ As a description of the next music director of an orchestra that has - 33 - hitherto been led by musicians like Gustav Mahler and Pierre Boulez, that seems likely to have struck at least some Times readers as faint praise. For my part, I have no idea whether Gilbert is a great conductor or even a good one. To be sure, he performs an impressive variety of interesting compositions, but it is not necessary for me to visit Avery Fisher Hall, or anywhere else, to hear interesting orchestral music. All I have to do is to go to my CD shelf, or boot up my computer and download still more recorded music from iTunes. Devoted concertgoers who reply that recordings are no substitute for live performance are missing the point. For the time, attention, and money of the art-loving public, classical instrumentalists must compete not only with opera houses, dance troupes, theater companies, and museums, but also with the recorded performances of the great classical musicians of the 20th century. There recordings are cheap, available everywhere, and very often much higher in artistic quality than today‘s live performances; moreover, they can be ,consumed‖ at a time and place of the listener’s choosing. The widespread availability of such recordings has thus brought about a crisis in the institution of the traditional classical concert. One possible response is for classical performers to program attractive new music that is not yet available on record. Gilbert‘s own interest in new music has been widely noted: Alex Ross, a classical-music critic, has described him as a man who is capable of turning the Philharmonic into ,a markedly different, more vibrant organization.‖ But what will be the nature of that difference? Merely expanding the orchestra‘s repertoire will not be enough. If Gilbert and the Philharmonic are to succeed, they must first change the relationship between America‘s oldest orchestra and the new audience it hops to attract. 21. We learn from Para.1 that Gilbert‘s appointment has [A]incurred criticism.[B]raised suspicion.[C]received acclaim.[D]aroused curiosity. 22. Tommasini regards Gilbert as an artist who is [A]influential.[B]modest.[C]respectable.[D]talented. 23. The author believes that the devoted concertgoers [A]ignore the expenses of live performances.[B]reject most kinds of recorded performances. [C]exaggerate the variety of live performances.[D]overestimate the value of live performances. 24. According to the text, which of the following is true of recordings? [A]They are often inferior to live concerts in quality.[B]They are easily accessible to the general public. [C]They help improve the quality of music.[D]They have only covered masterpieces. 25. Regarding Gilbert‘s role in revitalizing the Philharmonic, the author feels [A]doubtful.[B]enthusiastic.[C]confident.[D]puzzled. Text 2 When Liam McGee departed as president of Bank of America in August, his explanation was surprisingly straight up. Rather than cloaking his exit in the usual vague excuses, he came right out and said he was leaving ,to pursue my goal of running a company.‖ Broadcasting his ambition was ,very much my decision,‖ McGee says. Within two weeks, he was talking for the first time with the board of Hartford Financial Services Group, which named him CEO and chairman on September 29. McGee says leaving without a position lined up gave him time to reflect on what kind of company he wanted to run. It also sent a clear message to the outside world about his aspirations. And McGee isn‘t alone. In recent weeks the No.2 executives at Avon and American Express quit with the explanation that they were looking for a CEO post. As boards scrutinize succession plans in response to shareholder pressure, executives who don‘t get the nod also may wish to move on. A turbulent business environment also has senior managers cautious of letting vague pronouncements cloud their reputations. As the first signs of recovery begin to take hold, deputy chiefs may be more willing to make the jump without a net. - 34 - In the third quarter, CEO turnover was down 23% from a year ago as nervous boards stuck with the leaders they had, according to Liberum Research. As the economy picks up, opportunities will abound for aspiring leaders. The decision to quit a senior position to look for a better one is unconventional. For years executives and headhunters have adhered to the rule that the most attractive CEO candidates are the ones who must be senior partner Dennis Carey:‖I can‘t think of a single search I‘ve done where a board has not instructed me to look at sitting CEOs first.‖ Those who jumped without a job haven‘t always landed in top positions quickly. Ellen Marram quit as chief of Tropicana a decade age, saying she wanted to be a CEO. It was a year before she became head of a tiny Internet-based commodities exchange. Robert Willumstad left Citigroup in 2005 with ambitions to be a CEO. He finally took that post at a major financial institution three years later. Many recruiters say the old disgrace is fading for top performers. The financial crisis has made it more acceptable to be between jobs or to leave a bad one. ,The traditional rule was it‘s safer to stay where you are, but that‘s been fundamentally inverted,‖ says one headhunter. ,The people who‘ve been hurt the worst are those who‘ve stayed too long.‖ 26. When McGee announced his departure, his manner can best be described as being [A]arrogant.[B]frank.[C]self-centered.[D]impulsive. 27. According to Paragraph 2, senior executives‘ quitting may be spurred by [A]their expectation of better financial status.[B]their need to reflect on their private life. [C]their strained relations with the boards.[D]their pursuit of new career goals. 28. The word ,poached‖ (Line 3, Paragraph 4) most probably means [A]approved of.[B]attended to.[C]hunted for.[D]guarded against. 29. It can be inferred from the last paragraph that [A]top performers used to cling to their posts.[B]loyalty of top performers is getting out-dated. [C]top performers care more about reputations.[D]it‘s safer to stick to the traditional rules. 30. Which of the following is the best title for the text? [A]CEOs: Where to Go? [B]CEOs: All the Way Up? [C]Top Managers Jump without a Net [D]The Only Way Out for Top Performers Text 3 The rough guide to marketing success used to be that you got what you paid for. No longer. While traditional ,paid‖ media – such as television commercials and print advertisements – still play a major role, companies today can exploit many alternative forms of media. Consumers passionate about a product may create ,owned‖ media by sending e-mail alerts about products and sales to customers registered with its Web site. The way consumers now approach the broad range of factors beyond conventional paid media. Paid and owned media are controlled by marketers promoting their own products. For earned media , such marketers act as the initiator for users‘ responses. But in some cases, one marketer‘s owned media become another marketer‘s paid media – for instance, when an e-commerce retailer sells ad space on its Web site. We define such sold media as owned media whose traffic is so strong that other organizations place their content or e-commerce engines within that environment. This trend ,which we believe is still in its infancy, effectively began with retailers and travel providers such as airlines and hotels and will no doubt go further. Johnson & Johnson, for example, has created BabyCenter, a stand-alone media property that promotes complementary and even competitive products. Besides generating income, the presence of other marketers makes the site seem objective, gives companies opportunities to learn valuable information about the appeal of other companies‘ marketing, and may help expand user traffic for all companies concerned. The same dramatic technological changes that have provided marketers with more (and more diverse) communications choices have also increased the risk that passionate consumers will voice their opinions in quicker, more visible, and much more damaging ways. Such hijacked media are the opposite of earned media: an asset or campaign becomes hostage to consumers, other stakeholders, or activists who make negative allegations about a brand or product. - 35 - Members of social networks, for instance, are learning that they can hijack media to apply pressure on the businesses that originally created them. If that happens, passionate consumers would try to persuade others to boycott products, putting the reputation of the target company at risk. In such a case, the company‘s response may not be sufficiently quick or thoughtful, and the learning curve has been steep. Toyota Motor, for example, alleviated some of the damage from its recall crisis earlier this year with a relatively quick and well-orchestrated social-media response campaign, which included efforts to engage with consumers directly on sites such as Twitter and the social-news site Digg. 31.Consumers may create ,earned‖ media when they are [A] obsessed with online shopping at certain Web sites. [B] inspired by product-promoting e-mails sent to them. [C] eager to help their friends promote quality products.[D] enthusiastic about recommending their favorite products. 32. According to Paragraph 2,sold media feature [A] a safe business environment.[B] random competition. [C] strong user traffic. [D] flexibility in organization. 33. The author indicates in Paragraph 3 that earned media [A] invite constant conflicts with passionate consumers. [B] can be used to produce negative effects in marketing. [C] may be responsible for fiercer competition.[D] deserve all the negative comments about them. 34. Toyota Motor‘s experience is cited as an example of [A] responding effectively to hijacked media.[B] persuading customers into boycotting products. [C] cooperating with supportive consumers.[D] taking advantage of hijacked media. 35. Which of the following is the text mainly about ? [A] Alternatives to conventional paid media.[B] Conflict between hijacked and earned media. [C] Dominance of hijacked media.[D] Popularity of owned media. Text 4 It‘s no surprise that Jennifer Senior‘s insightful, provocative magazine cover story, ,I love My Children, I Hate My Life,‖ is arousing much chatter – nothing gets people talking like the suggestion that child rearing is anything less than a completely fulfilling, life-enriching experience. Rather than concluding that children make parents either happy or miserable, Senior suggests we need to redefine happiness: instead of thinking of it as something that can be measured by moment-to-moment joy, we should consider being happy as a past-tense condition. Even though the day-to-day experience of raising kids can be soul-crushingly hard, Senior writes that ,the very things that in the moment dampen our moods can later be sources of intense gratification and delight.‖ The magazine cover showing an attractive mother holding a cute baby is hardly the only Madonna-and-child image on newsstands this week. There are also stories about newly adoptive – and newly single – mom Sandra Bullock, as well as the usual ,Jennifer Aniston is pregnant‖ news. Practically every week features at least one celebrity mom, or mom-to-be, smiling on the newsstands. In a society that so persistently celebrates procreation, is it any wonder that admitting you regret having children is equivalent to admitting you support kitten-killing ? It doesn‘t seem quite fair, then, to compare the regrets of parents to the regrets of the children. Unhappy parents rarely are provoked to wonder if they shouldn‘t have had kids, but unhappy childless folks are bothered with the message that children are the single most important thing in the world: obviously their misery must be a direct result of the gaping baby-size holes in their lives. Of course, the image of parenthood that celebrity magazines like Us Weekly and People present is hugely unrealistic, especially when the parents are single mothers like Bullock. According to several studies concluding that parents are less happy than childless couples, single parents are the least happy of all. No shock there, considering how much work it is to raise a kid without a partner to lean on; yet to hear Sandra and Britney tell it, raising a kid on their ,own‖ (read: with round-the-clock help) is a piece of cake. It‘s hard to imagine that many people are dumb enough to want children just because Reese and Angelina make it look so glamorous: most adults understand that a baby is not a haircut. But it‘s interesting to wonder if the images we see every week of stress-free, happiness-enhancing parenthood aren‘t in some small, subconscious way contributing to our - 36 - own dissatisfactions with the actual experience, in the same way that a small part of us hoped getting , the Rachel‖ might make us look just a little bit like Jennifer Aniston. 36.Jennifer Senior suggests in her article that raising a child can bring [A]temporary delight[B]enjoyment in progress[C]happiness in retrospect[D]lasting reward 37.We learn from Paragraph 2 that [A]celebrity moms are a permanent source for gossip.[B]single mothers with babies deserve greater attention. [C]news about pregnant celebrities is entertaining.[D]having children is highly valued by the public. 38.It is suggested in Paragraph 3 that childless folks [A]are constantly exposed to criticism.[B]are largely ignored by the media. [C]fail to fulfill their social responsibilities.[D]are less likely to be satisfied with their life. 39.According to Paragraph 4, the message conveyed by celebrity magazines is [A]soothing.[B]ambiguous.[C]compensatory.[D]misleading. 40.Which of the following can be inferred from the last paragraph? [A]Having children contributes little to the glamour of celebrity moms. [B]Celebrity moms have influenced our attitude towards child rearing. [C]Having children intensifies our dissatisfaction with life. [D]We sometimes neglect the happiness from child rearing. Part B Directions: The following paragraph are given in a wrong order. For Questions 41-45, you are required to reorganize these paragraphs into a coherent text by choosing from the list A-G to filling them into the numbered boxes. Paragraphs E and G have been correctly placed. Mark your answers on ANSWER SHEET 1. (10 points) [A] No disciplines have seized on professionalism with as much enthusiasm as the humanities. You can, Mr Menand points out, became a lawyer in three years and a medical doctor in four. But the regular time it takes to get a doctoral degree in the humanities is nine years. Not surprisingly, up to half of all doctoral students in English drop out before getting their degrees. [B] His concern is mainly with the humanities: Literature, languages, philosophy and so on. These are disciplines that are going out of style: 22% of American college graduates now major in business compared with only 2% in history and 4% in English. However, many leading American universities want their undergraduates to have a grounding in the basic canon of ideas that every educated person should posses. But most find it difficult to agree on what a ,general education‖ should look like. At Harvard, Mr Menand notes, ,the great books are read because they have been read‖ -they form a sort of social glue. [C] Equally unsurprisingly, only about half end up with professorships for which they entered graduate school. There are simply too few posts. This is partly because universities continue to produce ever more PhDs. But fewer students want to study humanities subjects: English departments awarded more bachelor‘s degrees in 1970-71 than they did 20 years later. Fewer students requires fewer teachers. So, at the end of a decade of theses-writing, many humanities students leave the profession to do something for which they have not been trained. [D] One reason why it is hard to design and teach such courses is that they can cut across the insistence by top American universities that liberal-arts educations and professional education should be kept separate, taught in different schools. Many students experience both varieties. Although more than half of Harvard undergraduates end up in law, medicine or business, future doctors and lawyers must study a non-specialist liberal-arts degree before embarking on a professional qualification. [E] Besides professionalizing the professions by this separation, top American universities have professionalised the professor. The growth in public money for academic research has speeded the process: federal research grants rose fourfold between 1960and 1990, but faculty teaching hours fell by half as - 37 - research took its toll. Professionalism has turned the acquisition of a doctoral degree into a prerequisite for a successful academic career: as late as 1969a third of American professors did not possess one. But the key idea behind professionalisation, argues Mr Menand, is that ,the knowledge and skills needed for a particular specialization are transmissible but not transferable.‖So disciplines acquire a monopoly not just over the production of knowledge, but also over the production of the producers of knowledge. [F] The key to reforming higher education, concludes Mr Menand, is to alter the way in which ,the producers of knowledge are produced.‖Otherwise, academics will continue to think dangerously alike, increasingly detached from the societies which they study, investigate and criticize.‖Academic inquiry, at least in some fields, may need to become less exclusionary and more holistic.‖Yet quite how that happens, Mr Menand dose not say. [G] The subtle and intelligent little book The Marketplace of Ideas: Reform and Resistance in the American University should be read by every student thinking of applying to take a doctoral degree. They may then decide to go elsewhere. For something curious has been happening in American Universities, and Louis Menand, a professor of English at Harvard University, captured it skillfully. Part C Directions: Read the following text carefully and then translate the underlined segments into Chinese. Your translation should be written carefully on ANSWER SHEET 2. (10 points) With its theme that ,Mind is the master weaver,‖ creating our inner character and outer circumstances, the book As a Man Thinking by James Allen is an in-depth exploration of the central idea of self-help writing. thoughts can be hidden and made powerless; this allows us to think one way and act another. However, Allen believed Since desire and will are damaged by the presence of thoughts that do not accord with desire, Allen concluded : , We do not attract what we want, but what we are.‖ Achievement happens because you as a person embody the external achievement; you don‘t , get‖ success but become it. There is no gap between mind and matter. \Part of the fame of Allen‘s book is its contention that ,Circumstances do not make a person, they reveal him.‖This ,however, would be a knee-jerk reaction to a subtle argument. Each set of circumstances, however bad, offers a unique opportunity for growth. If circumstances always determined the life and prospects of people, then humanity would never have progressed. ‘s early life and its conditions are often the greatest gift to an individual. The sobering aspect of Allen‘s book is that we have no one else to blame for our present condition except ourselves. - 38 - 2011年考研英语真题答 - 39 - 2011年考研英语真题答 40 附阅读Part A翻译: 译文 1 2009年纽约交响乐团突然宣布聘用艾伦•吉尔伯特为下一位乐曲指挥,从那时起一直到现在,这次任命都成为古典音乐界的话题。退一步说,从总体上看,反应还是不错的。如冷静的古典音乐评论家安东尼•托姆西尼就这样写:从长时间来看,这次委命是英明的。 然而,这次任命还是令人意外。原因之一在于吉乐伯特名声相对较小。就连那时主张雇用吉尔伯特的托姆西尼,也称吉尔伯特其貌不扬,缺乏一位令人敬仰的指挥大师的气质。作为对这个很牛的管弦乐队(牛的表现:到目前为止一直被牛人领导着)下一任指挥家的描述,这种描述跟虚浮的赞扬一样,确实会令至少一部分泰晤士报的读者觉得愕然不解(让他们觉得不可思议)。 就我的观点而言,我不知道吉尔伯特是不是一位伟大的指挥家,甚至连他是不是算好的指挥家也不敢确定。可以确信的是,虽然他演出了很多令人印象深刻的有趣的乐曲。然而,我不需要访问Avery Fisher Hall(可能是纽约交响乐团所在地,即吉尔伯特表演之所),或者其他地方才能听到有趣的管弦乐。(作者意思是,不需要听吉尔伯特,到处可以听到有趣的管弦乐。)我所做的,只需要到我的CD棚里去,随便打开我的电脑,从ITUNES上就可下载比那(当指吉尔伯特表演的)多得多的类似的音乐。 对于唱片,那些专门参加音乐会的人会说,现场表演是不可替代的。他们显然忽视了一个要点。为了替音乐爱好者节省时间、精力、金钱考虑,古典乐曲表演表不仅要在各种表演场所进行竞争,还要在记录这些行为的媒介上竞争。记在唱片上的表演比现场表演更便宜,更易得,甚至质量更好。而且它们的消费时间地点可以任由听者选择。因此,这种唱片的广泛应用,给传统音乐会带来了生存危机。 一个可能的应对方式(解决办法)是古典音乐表演者发明有吸引力的从唱片上听不到的曲子。吉尔伯特在新音乐方面投入了自己的兴趣,这已广被人知:如古典音乐评论家罗斯就把吉尔伯特描述成一个可以扭转交响乐方向的人,认为他把交响乐带进了一个明显不同的更有活力的天地。但是,这种,不同‖的实质是什么呢,仅仅扩展交响乐的节目是不够的。吉尔伯特和交响乐要想取得成功,必须首先改变美国旧的管弦乐和它们想吸引的新的听众之间的关系。 译文 2 当列姆•麦克杰八月份从美国银行任上离职时,他的解释确实令人意外。与通常会用的模糊理由不同的是,他直率地说,他离开是为了找一家公司当管理者,而那是他一向就有的追求。他说,作出这一选择纯属个人原因。两周之 40 2011年考研英语真题答 41 行官和猎头们都坚持认为,最好的CEO候选人需要去挖别人的墙角才能得到(而不是那些主动离开原岗位的人)。某某猎头说,当董事会还没有委托我先去找一个还在任上的CEO时,我不能去考虑那些我在网上一搜就有的人。 那些没有工作去向就跳槽的人不会总是很快找到理想岗位。十年前爱伦•马拉姆从T公司领导人的位子上退下,也是为了当一个CEO。一年前她才成为一家小型电子交易所的领导人。罗伯特在2005年为了当CEO而离开,他最终在一家重要的财务机构找到这种工作是在三年之后。 许多招聘人表示,对于那些最好的演员来说,旧的耻辱正在淡忘。财务危机使得在两个工作机会之间进行选择或者离开更坏的工作这样的行为变得可以接受。,传统规则是,最好呆在你原来的地方,但现在这种规则被从根本上颠覆了。‖ 一个猎头说,,在一个地方呆得越久,就越容易受损。‖ 译文3 在过去,销售成功的基本法则是:种瓜得瓜,一分耕耘一分收获。现在不同了。传统的付出方式(媒介,指企业付钱给电视台做广告或者报社做报刊广告)——电视购物和印刷广告——虽然仍占主要地位,但是现在的企业可以开发出更多的替代这些媒介的形式。对产品有热情的用户可能通过给在自己网站上注册的顾客发关于产品和商品的电邮的提醒,来建立自己的媒介。这样用户现在接近了广阔的媒介因素,这些因素超越了传统的付费媒介。 付费并占有的媒介,是被想促销自己产品的商人控制的。而对于白捡的媒介(免费的媒介报道)而言,这种商人的角色仅是作为响应用户需求的第一环(直接面对用户的不是他们)。但是在一些案例中,一个商人拥有的媒介成为另一个商人的付费媒介(但有时候,促销产品的商人也直接面对用户,即把别人占有的媒介暂时变成自己占有的媒介)。例如,当一个电子商务零售商在自己的网站上出售广告空间时,就是如此。我们把这种出售的媒介定义为拥有的媒介。这种(出售空间式的)拥有的媒介是如此强大普遍,以致于其他团体把他们的希望(满意;内容;电子商务发动机)寄托在这种环境中。这种(寄托)趋势虽然依然在婴儿期,但我们相信这种从零售商和旅行提供商(如航空公司、旅馆)有效起步的趋势会越来越强劲。例如强生建立了一个婴儿中心,这是一种杰出的媒介资产,可用于推销提升配套产品,包括那些有竞争力的产品。除了带来利润,除了由于其他商人的到场可以使这个地方显得客观可信,以及给各个公司有机会了解有关其他公司需求的有价值的信息,还能有利于拓展所有公司都关心的用户交易。 这类戏剧性的技术革新给商人带来数量越来越多(种类也越来越多)的通信机会的同时,也同样会提高风险。因为热情的用户会更快、更形象、更有破坏力地表达自己的反对,这种被绑架的媒介,与上述的免费利用的媒介背道而驰(不是商人所希望出现的)。此时,媒介就像人质一样,成为敌人可用的财产或者发起的一次行动(敌人有用户、其他竞争对手、对某商标或产品向来没有好话的社会活动家)。例如,社会网络中的成员正在意识到他们可以绑架媒介,来对建立媒介的那些商人施加压力。 如果这种事情发生了,热情的用户就会努力劝说其他人抵制产品,使得目标公司声名处于危险之中。此时,公司的反应往往不会足够快,也不会足够理性,学习曲线将会变得很陡(学习曲线是表示单位产品生产时间与所生产的产品总数量之间的关系的一条曲线。一般情况下,产品总量越大,单个产品生产时间越短。也可以表示工人一定时间所犯错误数量与练习时间的关系,一般练习时间越长,单位时间内错误越少。这里的曲线陡时大约表示相同产量规模下,现在比原来单 个产品所消耗的资源更多,或者说相同练习程度下,一定时间错误更多。总之,是比原来更糟糕了)。例如丰田汽车,今年早些时候通过相对来说较快和精心策划的行动从车辆召回危机中把损尽量降下来,丰田的行动包括努力请用户进土威特这样的地方,挖掘社会新闻的利用等等。 41 42 译文 4 毫无疑问,作为有煸动性的杂志封面故事,詹尼弗西尼尔的深刻见解——,我爱我的孩子们,我讨厌我目前的生活状况‖——可以唤起人们的谈兴。可是,人们不会想到,养孩子可不是一件完全令人愉悦、生活充实的事情。西尼尔并没有简单地说,孩子使得父母既快乐又痛苦。她建议,我们需要重新定义幸福:幸福不应该像过去那样被定义为由一个个瞬间的快乐组合而成的东西;我们应该把幸福视为一种过去的状态。尽管抚养孩子的日子漫长难熬,令人筋疲力尽,但是西尼尔认为,正是那些心绪沉重的时刻,日后却给我们带来由衷的欣喜。 杂志封面上一位有魅力的母亲抱着一个可爱的婴儿,这种圣母与圣子的图画这周在报摊上可不止西尼尔这一起。例如杂志上讲到最近刚收养孩子的母亲——有时是刚变成单身母亲的人——桑德拉布鲁克,以及那种很常见的,詹尼弗阿尼斯顿怀孕了‖的新闻。实际上,每周都有至少一位名流母亲、或者准母亲在杂志上笑迎读者。 在一个坚持不懈地倡导生育的社会中,承认自己后悔生育孩子就相当于承认自己赞同谋杀宠物猫,这难道不值得反思吗,把父母亲的后悔与孩子的后悔相提并论(可能指把作为孩子家长的那种辛苦产生的悔恨理解为根源出在孩子身上,从而产生关于生下孩子的后悔),这显然并不合理。(因此)不情愿养孩子的父母很少会反思自己是否应该养育孩子。但是那不幸福的无孩子的人却为类似,孩子是世上唯一最可珍惜的东西‖这样的信息所烦恼。显然,他们的不幸必须通过生儿育女才能得以消除。 当然,在美国周刊与人这样的杂志上所提供的,社会名流父母亲‖现象是不切实际的。特别是当,父母亲‖是布鲁克这样的单身母亲时更是如此。多项研究表明,有孩子的父母很少比没有孩子的夫妇更快乐,而单亲家庭中的家长烦愁尤甚。这并不奇怪,因为一个人养一个孩子实在太麻烦了。然而,你看看桑德拉和布列尼说的话:自己,一个人‖养孩子,其实非常简单。(她们当然觉得简单了,因为她们是在周围人全天候的帮助下养着孩子的。) 当然,要说很多人傻头傻脑地生育孩子,只是因为里斯和安格丽娜这种名流使这种行为看上去显得诱人,这也是不可能的——多数成年人其实理解:养孩子可不是像做个发型那么简单。但是这确实是一件很有趣的值得反思的事情:我们每周看的,轻松快乐做父母‖的杂志封面,并不是通过潜意识的方式里让我们对(没有孩子的)现实经历不满,而是这些图片在潜意识中让我们有那种想成为雷切尔的心理,但实际上却使得我们看上去有点像詹尼弗亚尼斯顿。(大约指雷切尔养孩子显得潇洒,而詹尼弗生养孩子显得狼狈。) 42 2010年全国硕士研究生入学统一考试英语试题 43 2010年全国硕士研究生入学统一考试英语试题 Section I Use of English Directions: Read the following text. Choose the best word(s) for each numbered blank and mark [A], [B], [C] or [D] on ANSWER SHEET 1. (10 points) In 1924 America‘s National Research Council sent two engineers to supervise a series of industrial experiments at a large telephone-parts factory called the Hawthorne Plant near Chicago. It hoped they would learn how stop-floor lighting 1 workers‘ productivity. Instead, the studies ended 2 giving their name to the "Hawthorne effect", the extremely influential idea that the very 3 to being experimented upon changed subjects‘ behavior. The idea arose because of the 4 behavior of the women in the Hawthorne plant. According to 5 of the experiments, their hourly output rose when lighting was increased, but also when it was dimmed. It did not 6 what was done in the experiment; 7 something was changed, productivity rose. A(n) 8 that they were being experimented upon seemed to be 9 to alter workers‘ behavior 10 itself. After several decades, the same data were 11 to econometric the analysis. Hawthorne experiments has another surprise store 12the descriptions on record, no systematic 13 was found that levels of productivity were related to changes in lighting. It turns out that peculiar way of conducting the experiments may be have let to 14 interpretation of what happed. 15, lighting was always changed on a Sunday. When work started again on Monday, output 16 rose compared with the previous Saturday and 17 to rise for the next couple of days. 18, a comparison with data for weeks when there was no experimentation showed that output always went up on Monday, workers 19 to be diligent for the first few days of the week in any case, before 20 a plateau and then slackening off. This suggests that the alleged "Hawthorne effect" is hard to pin down. 1. [A] affected 2. [A] at [B] achieved [B] up [C] extracted [C] with 43 [D] restored [D] off 3. [A] truth 4. [A] controversial 5. [A] requirements 6. [A] conclude 7. [A] as far as 8. [A] awareness 9. [A] suitable 10. [A] about 11. [A] compared 12. [A] contrary to 13. [A] evidence 14. [A] disputable 15. [A] In contrast 16. [A] duly 18. [A] Therefore 19.[A] attempted 2010年全国硕士研究生入学统一考试英语试题 44 [B] sight [B] perplexing [B] explanations [B] matter [B] for fear that [B] expectation [B] excessive [B] for [B] shown [B] consistent with [B] guidance [B] enlightening [B] For example [B] accidentally [B] Furthermore [B] tended [C] act [C] mischievous [C] accounts [C] indicate [C] in case that [C] sentiment [C] enough [C] on [C] subjected [C] parallel with [C] implication [C] reliable [D] proof [D] ambiguous [D] assessments [D] work [D] so long as [D] illusion [D] abundant [D] by [D] conveyed [D] peculiar to [D] source [D] misleading [C] In consequence [D] As usual [C] unpredictably [C] However [C] chose [D] suddenly [D] Meanwhile [D] intended 20. [A] breaking [B] climbing [C] surpassing [D] hitting Section II Reading Comprehension Part A Directions: Read the following four texts. Answer the questions below each text by choosing [A], [B], [C] or [D]. Mark your answers on ANSWER SHEET 1. (40 points) Text 1 44 2010年全国硕士研究生入学统一考试英语试题 45 Of all the changes that have taken place in English-language newspapers during the past quarter-century, perhaps the most far-reaching has been the inexorable decline in the scope and seriousness of their arts coverage. It is difficult to the point of impossibility for the average reader under the age of forty to imagine a time when high-quality arts criticism could be found in most big-city newspapers. Yet a considerable number of the most significant collections of criticism published in the 20th century consisted in large part of newspaper reviews. To read such books today is to marvel at the fact that their learned contents were once deemed suitable for publication in general-circulation dailies. We are even farther removed from the unfocused newspaper reviews published in England between the turn of the 20th century and the eve of World War II, at a time when newsprint was dirt-cheap and stylish arts criticism was considered an ornament to the publications in which it appeared. In those far-off days, it was taken for granted that the critics of major papers would write in detail and at length about the events they covered. Theirs was a serious business, and even those reviewers who wore their learning lightly, like George Bernard Shaw and Ernest Newman, could be trusted to know what they were about. These men believed in journalism as a calling, and were proud to be published in the daily press. ,So few authors have brains enough or literary gift enough to keep their own end up in journalism,‖ Newman wrote, ,that I am tempted to define ,journalism‘ as ,a term of contempt applied by writers who are not read to writers who are.‘‖ Unfortunately, these critics are virtually forgotten. Neville Cardus, who wrote for the Manchester Guardian from 1917 until shortly before his death in 1975, is now known solely as a writer of essays on the game of cricket. During his lifetime, though, he was also one of England‘s foremost classical-music critics, a stylist so widely admired that his Autobiography (1947) became a best-seller. He was knighted in 1967, the first music critic to be so honored. Yet only one of his books is now in print, and his vast body of writings on music is unknown save to specialists. Is there any chance that Cardus‘s criticism will enjoy a revival? The prospect seems remote. Journalistic tastes had changed long before his death, and postmodern readers have little use for the richly upholstered Vicwardian prose in which he specialized. Moreover, the amateur tradition in music criticism has been in headlong retreat. 21. It is indicated in Paragraphs 1 and 2 that [A] arts criticism has disappeared from big-city newspapers. [B] English-language newspapers used to carry more arts reviews. [C] high-quality newspapers retain a large body of readers. [D] young readers doubt the suitability of criticism on dailies. 22. Newspaper reviews in England before World War II were characterized by [A] free themes.[B] casual style.[C] elaborate layout.[D] radical viewpoints. 23. Which of the following would Shaw and Newman most probably agree on? [A] It is writers‘ duty to fulfill journalistic goals. [B] It is contemptible for writers to be journalists. 45 2010年全国硕士研究生入学统一考试英语试题 46 [C] Writers are likely to be tempted into journalism. [D] Not all writers are capable of journalistic writing. 24. What can be learned about Cardus according to the last two paragraphs? [A] His music criticism may not appeal to readers today. [B] His reputation as a music critic has long been in dispute. [C] His style caters largely to modern specialists. [D] His writings fail to follow the amateur tradition. 25. What would be the best title for the text? [A] Newspapers of the Good Old Days [B] The Lost Horizon in Newspapers [C] Mournful Decline of Journalism [D] Prominent Critics in Memory Text 2 Over the past decade, thousands of patents have been granted for what are called business methods. Amazon.com received one for its "one-click" online payment system. Merrill Lynch got legal protection for an asset allocation strategy. One inventor patented a technique for lifting a box. Now the nation‘s top patent court appears completely ready to scale back on business-method patents, which have been controversial ever since they were first authorized 10 years ago. In a move that has intellectual-property lawyers abuzz the U.S. court of Appeals for the federal circuit said it would use a particular case to conduct a broad review of business-method patents. In re Bilski, as the case is known , is "a very big deal", says Dennis D. Crouch of the University of Missouri School of law. It "has the potential to eliminate an entire class of patents." Curbs on business-method claims would be a dramatic about-face, because it was the federal circuit itself that introduced such patents with its 1998 decision in the so-called state Street Bank case, approving a patent on a way of pooling mutual-fund assets. That ruling produced an explosion in business-method patent filings, initially by emerging internet companies trying to stake out exclusive rights to specific types of online transactions. Later, more established companies raced to add such patents to their files, if only as a defensive move against rivals that might beat them to the punch. In 2005, IBM noted in a court filing that it had been issued more than 300 business-method patents despite the fact that it questioned the legal basis for granting them. Similarly, some Wall Street investment films armed themselves with patents for financial products, even as they took positions in court cases opposing the practice. The Bilski case involves a claimed patent on a method for hedging risk in the energy market. The Federal circuit issued an unusual order stating that the case would be heard by all 12 of the court‘s judges, rather than a typical panel of three, and that one issue it wants to evaluate is whether it should "reconsider" its state street Bank ruling. The Federal Circuit‘s action comes in the wake of a series of recent decisions by the supreme Court that has narrowed the scope of protections for patent holders. Last April, for example the justices signaled that too many patents were being upheld for "inventions" that are obvious. The judges on the Federal circuit are "reacting to the anti-patent trend at the Supreme Court", says 46 2010年全国硕士研究生入学统一考试英语试题 47 Harold C. Wegner, a patent attorney and professor at George Washington University Law School. 26. Business-method patents have recently aroused concern because of [A] their limited value to business [B] their connection with asset allocation [C] the possible restriction on their granting [D] the controversy over their authorization 27. Which of the following is true of the Bilski case? [A] Its ruling complies with the court decisions[B] It involves a very big business transaction [C] It has been dismissed by the Federal Circuit[D] It may change the legal practices in the U.S. 28. The word "about-face" (Line 1, Para 3) most probably means [A] loss of good will [B] increase of hostility [C] change of attitude [D] enhancement of dignity 29. We learn from the last two paragraphs that business-method patents [A] are immune to legal challenges [B] are often unnecessarily issued [C] lower the esteem for patent holders [D] increase the incidence of risks 30. Which of the following would be the subject of the text? [A] A looming threat to business-method patents [B] Protection for business-method patent holders [C] A legal case regarding business-method patents [D] A prevailing trend against business-method patents Text 3 In his book The Tipping Point, Malcolm Gladwell argues that social epidemics are driven in large part by the acting of a tiny minority of special individuals, often called influentials, who are unusually informed, persuasive, or well-connected. The idea is intuitively compelling, but it doesn‘t explain how ideas actually spread. The supposed importance of influentials derives from a plausible sounding but largely untested theory called the "two step flow of communication": Information flows from the media to the influentials and from them to everyone else. Marketers have embraced the two-step flow because it suggests that if they can just find and influence the influentials, those selected people will do most of the work for them. The theory also seems to explain the sudden and unexpected popularity of certain looks, brands, or neighborhoods. In many such cases, a cursory search for causes finds that some small group of people was wearing, promoting, or developing whatever it is before anyone else paid attention. Anecdotal evidence of this kind fits nicely with the idea that only certain special people can drive trends In their recent work, however, some researchers have come up with the finding that influentials have far less impact on social epidemics than is generally supposed. In fact, they don‘t seem to be required of all. 47 2010年全国硕士研究生入学统一考试英语试题 48 The researchers‘ argument stems from a simple observing about social influence, with the exception of a few celebrities like Oprah Winfrey—whose outsize presence is primarily a function of media, not interpersonal, influence—even the most influential members of a population simply don‘t interact with that many others. Yet it is precisely these non-celebrity influentials who, according to the two-step-flow theory, are supposed to drive social epidemics by influencing their friends and colleagues directly. For a social epidemic to occur, however, each person so affected, must then influence his or her own acquaintances, who must in turn influence theirs, and so on; and just how many others pay attention to each of these people has little to do with the initial influential. If people in the network just two degrees removed from the initial influential prove resistant, for example, the cascade of change won‘t propagate very far or affect many people. Building on the basic truth about interpersonal influence, the researchers studied the dynamics of populations manipulating a number of variables relating of populations, manipulating a number of variables relating to people‘s ability to influence others and their tendency to be influenced. Our work shows that the principal requirement for what we call "global cascades" – the widespread propagation of influence through networks – is the presence not of a few influentials but, rather, of a critical mass of easily influenced people. 31. By citing the book The Tipping Point, the author intends to [A] analyze the consequences of social epidemics [B] discuss influentials‘ function in spreading ideas [C] exemplify people‘s intuitive response to social epidemics [D] describe the essential characteristics of influentials. 32. The author suggests that the "two-step-flow theory" [A] serves as a solution to marketing problems[B] has helped explain certain prevalent trends [C] has won support from influentials[D] requires solid evidence for its validity 33. What the researchers have observed recently shows that [A] the power of influence goes with social interactions [B] interpersonal links can be enhanced through the media [C] influentials have more channels to reach the public [D] most celebrities enjoy wide media attention 34. The underlined phrase "these people" in paragraph 4 refers to the ones who [A] stay outside the network of social influence[B] have little contact with the source of influence [C] are influenced and then influence others[D] are influenced by the initial influential 35. what is the essential element in the dynamics of social influence? [A] The eagerness to be accepted[B] The impulse to influence others [C] The readiness to be influenced[D] The inclination to rely on others 48 2010年全国硕士研究生入学统一考试英语试题 49 Text 4 Bankers have been blaming themselves for their troubles in public. Behind the scenes, they have been taking aim at someone else: the accounting standard-setters. Their rules, moan the banks, have forced them to report enormous losses, and it‘s just not fair. These rules say they must value some assets at the price a third party would pay, not the price managers and regulators would like them to fetch. Unfortunately, banks‘ lobbying now seems to be working. The details may be unknowable, but the independence of standard-setters, essential to the proper functioning of capital markets, is being compromised. And, unless banks carry toxic assets at prices that attract buyers, reviving the banking system will be difficult. After a bruising encounter with Congress, America‘s Financial Accounting Standards Board (FASB) rushed through rule changes. These gave banks more freedom to use models to value illiquid assets and more flexibility in recognizing losses on long-term assets in their income statement. Bob Herz, the FASB‘s chairman, cried out against those who "question our motives." Yet bank shares rose and the changes enhance what one lobby group politely calls "the use of judgment by management." European ministers instantly demanded that the International Accounting Standards Board (IASB) do likewise. The IASB says it does not want to act without overall planning, but the pressure to fold when it completes it reconstruction of rules later this year is strong. Charlie McCreevy, a European commissioner, warned the IASB that it did "not live in a political vacuum" but "in the real word" and that Europe could yet develop different rules. It was banks that were on the wrong planet, with accounts that vastly overvalued assets. Today they argue that market prices overstate losses, because they largely reflect the temporary illiquidity of markets, not the likely extent of bad debts. The truth will not be known for years. But bank‘s shares trade below their book value, suggesting that investors are skeptical. And dead markets partly reflect the paralysis of banks which will not sell assets for fear of booking losses, yet are reluctant to buy all those supposed bargains. To get the system working again, losses must be recognized and dealt with. America‘s new plan to buy up toxic assets will not work unless banks mark assets to levels which buyers find attractive. Successful markets require independent and even combative standard-setters. The FASB and IASB have been exactly that, cleaning up rules on stock options and pensions, for example, against hostility from special interests. But by giving in to critics now they are inviting pressure to make more concessions. 36. Bankers complained that they were forced to [A] follow unfavorable asset evaluation rules [B] collect payments from third parties [C] cooperate with the price managers [D] reevaluate some of their assets. 37. According to the author , the rule changes of the FASB may result in [A] the diminishing role of management [B] the revival of the banking system 49 2010年全国硕士研究生入学统一考试英语试题 50 [C] the banks‘ long-term asset losses [D] the weakening of its independence 38. According to Paragraph 4, McCreevy objects to the IASB‘s attempt to [A] keep away from political influences. [B] evade the pressure from their peers. [C] act on their own in rule-setting. [D] take gradual measures in reform. 39. The author thinks the banks were "on the wrong planet" in that they [A] misinterpreted market price indicators [B] exaggerated the real value of their assets [C] neglected the likely existence of bad debts. [D] denied booking losses in their sale of assets. 40. The author‘s attitude towards standard-setters is one of [A] satisfaction. [B] skepticism. [C] objectiveness [D] sympathy Part B Directions: For Questions 41-45, choose the most suitable paragraphs from the list A-G and fill them into the numbered boxes to form a coherent text. Paragraph E has been correctly placed. There is one paragraph which does not fit in with the text. Mark your answers on ANSWER SHEET1. (10 points) [A] The first and more important is the consumer‘s growing preference for eating out; the consumption of food and drink in places other than homes has risen from about 32 percent of total consumption in 1995 to 35 percent in 2000 and is expected to approach 38 percent by 2005. This development is boosting wholesale demand from the food service segment by 4 to 5 percent a year across Europe, compared with growth in retail demand of 1 to 2 percent. Meanwhile, as the recession is looming large, people are getting anxious. They tend to keep a tighter hold on their purse and consider eating at home a realistic alternative. [B] Retail sales of food and drink in Europe‘s largest markets are at a standstill, leaving European grocery retailers hungry for opportunities to grow. Most leading retailers have already tried e-commerce, with limited success, and expansion abroad. But almost all have ignored the big, profitable opportunity in their own backyard: the wholesale food and drink trade, which appears to be just the kind of market retailers need. [C] Will such variations bring about a change in the overall structure of the food and drink market? Definitely not. The functioning of the market is based on flexible trends dominated by potential buyers. In other words, it is up to the buyer, rather than the seller, to decide what to buy .At any rate, this change will ultimately be acclaimed by an ever-growing number of both domestic and international consumers, regardless of how long the current consumer pattern will take hold. [D] All in all, this clearly seems to be a market in which big retailers could profitably apply their scale, existing infrastructure and proven skills in the management of product ranges, logistics, 50 2010年全国硕士研究生入学统一考试英语试题 51 and marketing intelligence. Retailers that master the intricacies of wholesaling in Europe may well expect to rake in substantial profits thereby. At least, that is how it looks as a whole. Closer inspection reveals important differences among the biggest national markets, especially in their customer segments and wholesale structures, as well as the competitive dynamics of individual food and drink categories. Big retailers must understand these differences before they can identify the segments of European wholesaling in which their particular abilities might unseat smaller but entrenched competitors. New skills and unfamiliar business models are needed too. [E] Despite variations in detail, wholesale markets in the countries that have been closely examined—France, Germany, Italy, and Spain—are made out of the same building blocks. Demand comes mainly from two sources: independent mom-and-pop grocery stores which, unlike large retail chains, are too small to buy straight from producers, and food service operators that cater to consumers when they don‘t eat at home. Such food service operators range from snack machines to large institutional catering ventures, but most of these businesses are known in the trade as "horeca": hotels, restaurants, and cafes. Overall, Europe‘s wholesale market for food and drink is growing at the same sluggish pace as the retail market, but the figures, when added together, mask two opposing trends. [F] For example, wholesale food and drink sales come to $268 billion in France, Germany, Italy, Spain, and the United Kingdom in 2000—more than 40 percent of retail sales. Moreover, average overall margins are higher in wholesale than in retail; wholesale demand from the food service sector is growing quickly as more Europeans eat out more often; and changes in the competitive dynamics of this fragmented industry are at last making it feasible for wholesalers to consolidate. [G] However, none of these requirements should deter large retailers (and even some large good producers and existing wholesalers) from trying their hand, for those that master the intricacies of wholesaling in Europe stand to reap considerable gains. E Part C Directions: Read the following text carefully and then translate the underlined segments into Chinese. Your translation should be written carefully on ANSWER SHEET 2. (10 points) One basic weakness in a conservation system based wholly on economic motives is that most members of the land community have no economic value. Yet these creatures are members of the biotic community and, if its stability depends on its integrity, they are entitled to continuance. When one of these noneconomic categories is threatened and, if we happen to love it .We invert excuses to give it economic importance. At the beginning of century songbirds were supposed order to be valid. 51 2010年考研英语真题答案 52 A parallel situation exists in respect of predatory mammals and fish-eating birds. (48) Some species of tree have been read out of the party by economics-minded foresters because they grow too slowly, or have too low a sale vale to pay as timber crops. (49) To sum up: a system of conservation based solely on economic self-interest is hopelessly lopsided. (50) falsely, I think, that the economic parts of the biotic clock will function without the uneconomic parts. 2010年考研英语真题答案 46.科学家们提出一些明显站不住脚的证据迅速来拯救,其大意是:如果鸟类无 法控制害虫,那么这些害虫就会吃光我们人类。 47. 但我们至少几乎也承认这样一点:不管鸟类是否给我们带来经济上的好处, 但鸟类作为生物其固有的权利应该继续存在。 48. 曾几何时,生物学家们有点过度使用这个证据,即这些物种通过杀死体质 弱者来保持猎物的正常繁衍或 者这些生物捕杀的仅仅是毫无价值的物种。 49. 在欧洲,林业在生态方面更加发达,无商业价值的树种被看作是原生森林 群落的一部分,而得到合理的保护。 50. 这一系统易于忽视,因而最终会消除掉这个土地共同体里的许多要素(成 员),虽然这些要素(成员)缺乏商业价值,但这些要素(成员)对这个共同体的健康 运行来说是必要的。 52 2009年全国硕士研究生入学统一考试英语试题 53 2009年全国硕士研究生入学统一考试英语试题 Directions: Read the following text. Choose the best word(s) for each numbered blank and mark A, B, C or D on ANSWER SHEET 1. (10 points) Research on animal intelligence always makes me wonder just how smart humans are. 1 the fruit-fly experiments described in Carl Zimmer‘s piece in the Science Times on Tuesday. Fruit flies who were taught to be smarter than the average fruit fly 2 to live shorter lives. This suggests that 3 bulbs burn longer, that there is an 4 in not being too terrifically bright. Intelligence, it 5 out, is a high-priced option. It takes more upkeep, burns more fuel and is slow 6 the starting line because it depends on learning — a gradual 7 — instead of instinct. Plenty of other species are able to learn, and one of the things they‘ve apparently learned is when to 8. Is there an adaptive value to 9 intelligence? That‘s the question behind this new research. I like it. Instead of casting a wistful glance 10 at all the species we‘ve left in the dust I.Q.-wise, it implicitly asks what the real 11 of our own intelligence might be. This is 12 the mind of every animal I‘ve ever met. Research on animal intelligence also makes me wonder what experiments animals would 13 on humans if they had the chance. Every cat with an owner, 14, is running a small-scale study in operant conditioning. we believe that 15 animals ran the labs, they would test us to 16 the limits of our patience, our faithfulness, our memory for terrain. They would try to decide what intelligence in humans is really 1, not merely how much of it there is. 19 question: Are humans actually aware of the Section I Use of English 53 2009年全国硕士研究生入学统一考试英语试题 54 world they live in? 20 the results are inconclusive. 1. [A] Suppose 2. [A] tended 3. [A] thinner 4. [A] tendency 5. [A] insists on 6. [A] off 7. [A] incredible 8. [A] fight 9. [A] invisible 10. [A] upward 11. [A] features 12. [A] outside 13. [A] deliver 14. [A] by chance 15. [A] if 16. [A] moderate 17. [A] at 18. [A] Above all 19. [A] fundamental 20. [A] By accident [B] Consider [C] Observe [D] Imagine [B] feared [C] happened [D] threatened [B] stabler [C] lighter [D] dimmer [B] advantage [C] inclination [D] priority [B] sums up [C] turns out [D] puts forward [B] behind [C] over [D] along [B] spontaneous [C] inevitable [D] gradual [B] doubt [C] stop [D] think [B] limited [C] indefinite [D] different [B] forward [C] afterward [D] backward [B] influences [C] results [D] costs [B] on [C] by [D] across [B] carry [C] perform [D] apply [B] in contrast [C] as usual [D] for instance [B] unless [C] as [D] lest [B] overcome [C] determine [D] reach [B] for [C] after [D] with [B] After all [C] However [D] Otherwise [B] comprehensive [C] equivalent [D] hostile [B] In time [C] So far [D] Better still Section II Reading Comprehension 54 Part A Directions: 2009年全国硕士研究生入学统一考试英语试题 55 Read the following four texts. Answer the questions below each text by choosing A, B, C or D. Mark your answers on ANSWER SHEET 1. (40 points) Text 1 Habits are a funny thing. We reach for them mindlessly, setting our brains on auto-pilot and relaxing into the unconscious comfort of familiar routine. "Not choice, but habit rules the unreflecting herd," William Wordsworth said in the 19th century. In the ever-changing 21st century, even the word "habit" carries a negative implication. So it seems paradoxical to talk about habits in the same context as creativity and innovation. But brain researchers have discovered that when we consciously develop new habits, we create parallel paths, and even entirely new brain cells, that can jump our trains of thought onto new, innovative tracks. Rather than dismissing ourselves as unchangeable creatures of habit, we can instead direct our own change by consciously developing new habits. In fact, the more new things we try—the more we step outside our comfort zone—the more inherently creative we become, both in the workplace and in our personal lives. But don‘t bother trying to kill off old habits; of procedure are worn into the hippocampus, they‘re there to stay. Instead, the new habits we deliberately ingrain into ourselves create parallel pathways that can bypass those old roads. "The first thing needed for innovation is a fascination with wonder," says Dawna Markova, author of "The Open Mind" and an executive change consultant for Professional Thinking Partners. "But we are taught instead to ?decide,‘ just as our president calls himself ?the Decider.‘ " She adds, however, that "to decide is to kill off all possibilities but one. A good innovational thinker is always exploring the many other possibilities." All of us work through problems in ways of which we‘re unaware, she says. Researchers in the late 1960s discovered that humans are born with the capacity to approach challenges in four primary ways: analytically, procedurally, relationally (or collaboratively) and innovatively. At the end of adolescence, however, the brain shuts down half of that capacity, preserving only those modes of thought that have seemed most valuable during the first decade or so of life. The current emphasis on standardized testing highlights analysis and procedure, meaning that few of us inherently use our innovative and collaborative modes of thought. "This breaks the major rule in the American belief system — that anyone can do anything," explains M. J. Ryan, author of the 2006 book This Year I Will... and Ms. Markova‘s business partner. "That‘s a lie that we have perpetuated, and it fosters commonness. Knowing what you‘re good at and doing even more of it creates excellence." This is where developing new habits comes in. 21. The Wordsworth‘s view, ,habits‖ is claimed by being ________. A. casual B. familiar C. mechanical D. changeable 55 2009年全国硕士研究生入学统一考试英语试题 56 22. Brain researchers have discovered that the formation of new habits can be ________ A. predicted B. regulated C. traced D. guided 23. The word "ruts"( Line 1, Paragraph 4) is closest meaning to ________ A. tracks B. series C. characteristics D. connections 24. Dawna Markova would most probably agree that ________. A. ideas are born of a relaxing mind B. innovativeness could be taught C. decisiveness derives from fantastic ideas D. curiosity activates creative minds 25. Ryan‘s comments suggest that the practice of standardized testing ________ A, prevents new habits form being formed B, no longer emphasizes commonness C, maintains the inherent American thinking model D, complies with the American belief system Text 2 It is a wise father that knows his own child, but today a man can boost his paternal (fatherly) wisdom – or at least confirm that he‘s the kid‘s dad. All he needs to do is shell our $30 for paternity testing kit (PTK) at his local drugstore – and another $120 to get the results. More than 60,000 people have purchased the PTKs since they first become available without prescriptions last years, according to Doug Fog, chief operating officer of Identigene, which makes the over-the-counter kits. More than two dozen companies sell DNA tests Directly to the public , ranging in price from a few hundred dollars to more than $2,500. Among the most popular: paternity and kinship testing , which adopted children can use to find their biological relatives and families can use to track down kids put up for adoption. DNA testing is also the latest rage among passionate genealogists—and supports businesses that offer to search for a family‘s geographic roots . Most tests require collecting cells by swabbing saliva in the mouth and sending it to the company for testing. All tests require a potential candidate with whom to compare DNA. But some observers are skeptical, "There is a kind of false precision being hawked by people claiming they are doing ancestry testing," says Trey Duster, a New York University sociologist. He notes that each individual has many ancestors-numbering in the hundreds just a few centuries back. Yet most ancestry testing only considers a single lineage, either the Y chromosome inherited through men in a father‘s line or mitochondrial DNA, which is passed down only from mothers. This DNA can reveal genetic information about only one or two ancestors, even though, for example, just three generations back people also have six other great-grandparents or, four generations back, 14 other great-great-grandparents. Critics also argue that commercial genetic testing is only as good as the reference collections to which a sample is compared. Databases used by some companies don‘t rely on data collected systematically but rather lump together information from different research projects. This means that a DNA database may have a lot of data from some regions and not others, so a person‘s test results may differ depending on the company that processes the results. In addition, the computer programs a company uses to estimate relationships may be patented and not subject to peer review 56 or outside evaluation. 2009年全国硕士研究生入学统一考试英语试题 57 26. In paragraphs 1 and 2, the text shows PTK‘s ___________. [A] easy availability[B] flexibility in pricing [C] successful promotion[D] popularity with households 27. PTK is used to __________. [A] locate one‘s birth place[B] promote genetic research [C] identify parent-child kinship[D] choose children for adoption 28. Skeptical observers believe that ancestry testing fails to__________. [A] trace distant ancestors[B] rebuild reliable bloodlines [C] fully use genetic information[D] achieve the claimed accuracy 29. In the last paragraph, a problem commercial genetic testing faces is __________. [A] disorganized data collection[B] overlapping database building [C] excessive sample comparison[D] lack of patent evaluation 30. An appropriate title for the text is most likely to be__________. [A] Fors and Againsts of DNA testing[B] DNA Testing and It‘s Problems [C] DNA Testing Outside the Lab[D] Lies Behind DNA Testing Text 3 The relationship between formal education and economic growth in poor countries is widely misunderstood by economists and politicians alike. Progress in both area is undoubtedly necessary for the social, political and intellectual development of these and all other societies; however, the conventional view that education should be one of the very highest priorities for promoting rapid economic development in poor countries is wrong. We are fortunate that is it, because building new educational systems there and putting enough people through them to improve economic performance would require two or three generations. The findings of a research institution have consistently shown that workers in all countries can be trained on the job to achieve radically higher productivity and, as a result, radically higher standards of living. Ironically, the first evidence for this idea appeared in the United States. Not long ago, with the country entering a recessing and Japan at its pre-bubble peak. The U.S. workforce was derided as poorly educated and one of the primary cause of the poor U.S. economic performance. Japan was, and remains, the global leader in automotive-assembly productivity. Yet the research revealed that the U.S. factories of Honda, Nissan, and Toyota achieved about 95 percent of the productivity of their Japanese counterparts -- a result of the training that U.S. workers received on the job. More recently, while examining housing construction, the researchers discovered that illiterate, non-English- speaking Mexican workers in Houston, Texas, consistently met best-practice labor productivity standards despite the complexity of the building industry‘s work. 57 2009年全国硕士研究生入学统一考试英语试题 58 What is the real relationship between education and economic development? We have to suspect that continuing economic growth promotes the development of education even when governments don‘t force it. After all, that‘s how education got started. When our ancestors were hunters and gatherers 10,000 years ago, they didn‘t have time to wonder much about anything besides finding food. Only when humanity began to get its food in a more productive way was there time for other things. As education improved, humanity‘s productivity potential increased as well. When the competitive environment pushed our ancestors to achieve that potential, they could in turn afford more education. This increasingly high level of education is probably a necessary, but not a sufficient, condition for the complex political systems required by advanced economic performance. Thus poor countries might not be able to escape their poverty traps without political changes that may be possible only with broader formal education. A lack of formal education, however, doesn‘t constrain the ability of the developing world‘s workforce to substantially improve productivity for the foreseeable future. On the contrary, constraints on improving productivity explain why education isn‘t developing more quickly there than it is. 31. The author holds in paragraph 1 that the important of education in poor countries ___________. [A] is subject groundless doubts[B] has fallen victim of bias [C] is conventional downgraded[D] has been overestimated 32. It is stated in Paragraph 1 that construction of a new education system __________. [A] challenges economists and politicians[B] takes efforts of generations [C] demands priority from the government[D] requires sufficient labor force 33. A major difference between the Japanese and U.S workforces is that __________. [A] the Japanese workforce is better disciplined[B] the Japanese workforce is more productive [C] the U.S workforce has a better education[D] the U.S workforce is more organize 34. The author quotes the example of our ancestors to show that education emerged __________. [A] when people had enough time[B] prior to better ways of finding food [C] when people on longer went hung[D] as a result of pressure on government 35. According to the last paragraph , development of education __________. [A] results directly from competitive environments [B] does not depend on economic performance [C] follows improved productivity[D] cannot afford political changes Text 4 The most thoroughly studied intellectuals in the history of the New World are the ministers and political leaders of seventeenth-century New England. According to the standard history of 58 2009年全国硕士研究生入学统一考试英语试题 59 American philosophy, nowhere else in colonial America was "So much importance attached to intellectual pursuits. " According to many books and articles, New England‘s leaders established the basic themes and preoccupations of an unfolding, dominant Puritan tradition in American intellectual life. To take this approach to the New Englanders normally means to start with the Puritans‘ theological innovations and their distinctive ideas about the church-important subjects that we may not neglect. But in keeping with our examination of southern intellectual life, we may consider the original Puritans as carriers of European culture, adjusting to New World circumstances. The New England colonies were the scenes of important episodes in the pursuit of widely understood ideals of civility and virtuosity. The early settlers of Massachusetts Bay included men of impressive education and influence in England. Besides the ninety or so learned ministers who came to Massachusetts church in the decade after 1629,There were political leaders like John Winthrop, an educated gentleman, lawyer, and official of the Crown before he journeyed to Boston. There men wrote and published extensively, reaching both New World and Old World audiences, and giving New England an atmosphere of intellectual earnestness. We should not forget , however, that most New Englanders were less well educated. While few craftsmen or farmers, let alone dependents and servants, left literary compositions to be analyzed, it is obvious that their views were less fully intellectualized. Their thinking often had a traditional superstitions quality. A tailor named John Dane, who emigrated in the late 1630s, left an account of his reasons for leaving England that is filled with signs. Sexual confusion, economic frustrations , and religious hope—all came together in a decisive moment when he opened the Bible, told his father the first line he saw would settle his fate, and read the magical words: "come out from among them, touch no unclean thing , and I will be your God and you shall be my people." One wonders what Dane thought of the careful sermons explaining the Bible that he heard in puritan churches. Meanwhile, many settles had slighter religious commitments than Dane‘s, as one clergyman learned in confronting folk along the coast who mocked that they had not come to the New world for religion . "Our main end was to catch fish. " 36. The author notes that in the seventeenth-century New England___________. [A] Puritan tradition dominated political life. [B] intellectual interests were encouraged. [C] Politics benefited much from intellectual endeavors. [D] intellectual pursuits enjoyed a liberal environment. 37. It is suggested in paragraph 2 that New Englanders__________. [A] experienced a comparatively peaceful early history. [B] brought with them the culture of the Old World [C] paid little attention to southern intellectual life [D] were obsessed with religious innovations 59 2009年全国硕士研究生入学统一考试英语试题 60 38. The early ministers and political leaders in Massachusetts Bay__________. [A] were famous in the New World for their writings [B] gained increasing importance in religious affairs [C] abandoned high positions before coming to the New World [D] created a new intellectual atmosphere in New England 39. The story of John Dane shows that less well-educated New Englanders were often __________. [A] influenced by superstitions[B] troubled with religious beliefs [C] puzzled by church sermons[D] frustrated with family earnings 40. The text suggests that early settlers in New England__________. [A] were mostly engaged in political activities[B] were motivated by an illusory prospect [C] came from different backgrounds.[D] left few formal records for later reference Part B Directions: Directions: In the following text, some sentences have been removed. For Questions (41-45), choose the most suitable one from the list A-G to fit into each of the numbered blank. There are two extra choices, which do not fit in any of the gaps. Mark your answers on ANSWER SHEET 1. (10 points) Coinciding with the groundbreaking theory of biological evolution proposed by British naturalist Charles Darwin in the 1860s, British social philosopher Herbert Spencer put forward his own theory of biological and cultural evolution. Spencer argued that all worldly phenomena, including human societies, changed over time, advancing toward perfection. 41.____________. American social scientist Lewis Henry Morgan introduced another theory of cultural evolution in the late 1800s. Morgan, along with Tylor, was one of the founders of modern anthropology. In his work, he attempted to show how all aspects of culture changed together in the evolution of societies.42._____________. In the early 1900s in North America, German-born American anthropologist Franz Boas developed a new theory of culture known as historical particularism. Historical particularism, which emphasized the uniqueness of all cultures, gave new direction to anthropology. 43._____________. Boas felt that the culture of any society must be understood as the result of a unique history and not as one of many cultures belonging to a broader evolutionary stage or type of culture. 44._______________. Historical particularism became a dominant approach to the study of culture in American anthropology, largely through the influence of many students of Boas. But a number of anthropologists in the early 1900s also rejected the particularist theory of culture in favor of diffusionism. Some attributed virtually every important cultural achievement to the inventions of a few, especially gifted peoples that, according to diffusionists, then spread to other cultures. 60 45.________________. 2009年全国硕士研究生入学统一考试英语试题 61 Also in the early 1900s, French sociologist Émile Durkheim developed a theory of culture that would greatly influence anthropology. Durkheim proposed that religious beliefs functioned to reinforce social solidarity. An interest in the relationship between the function of society and culture—known as functionalism—became a major theme in European, and especially British, anthropology. [A] Other anthropologists believed that cultural innovations, such as inventions, had a single origin and passed from society to society. This theory was known as diffusionism. [B] In order to study particular cultures as completely as possible, Boas became skilled in linguistics, the study of languages, and in physical anthropology, the study of human biology and anatomy. [C] He argued that human evolution was characterized by a struggle he called the "survival of the fittest," in which weaker races and societies must eventually be replaced by stronger, more advanced races and societies. [D] They also focused on important rituals that appeared to preserve a people‘s social structure, such as initiation ceremonies that formally signify children‘s entrance into adulthood. [E] Thus, in his view, diverse aspects of culture, such as the structure of families, forms of marriage, categories of kinship, ownership of property, forms of government, technology, and systems of food production, all changed as societies evolved. [F] Supporters of the theory viewed as a collection of integrated parts that work together to keep a society functioning. [G] For example, British anthropologists Grafton Elliot Smith and W. J. Perry incorrectly suggested, on the basis of inadequate information, that farming, pottery making, and metallurgy all originated in ancient Egypt and diffused throughout the world. In fact, all of these cultural developments occurred separately at different times in many parts of the world. Part C Directions: Read the following text carefully and then translate the underlined segments into Chinese. Your translation should be written carefully on ANSWER SHEET 2. (10 points) There is a marked difference between the education which everyone gets from living with others, and the deliberate educating of the young. In the former case the education is incidental; it is desire to secure the favor of overruling powers and to ward off evil influences; family life in the desire to gratify appetites and secure family perpetuity; systematic labor, for the most part, because the intellectual and emotional reaction of the forms of human association under which the world‘s 61 2009年考研英语真题答案 62 work is carried on receives little attention as compared with physical output. But in dealing with the young, the fact of association itself as an immediate human fact, gains pressure to accomplish a change in their attitude and habits is too urgent to leave these If humanity has made some headway in realizing that the ultimate value of every institution is its distinctively human effect we may well believe that this lesson has been learned largely through dealings with the young. 2009年考研英语真题答案 Part A (40 points) Part B (10 points) 46. 虽然我们可以说衡量任何一个社会机构价值的 标准 excel标准偏差excel标准偏差函数exl标准差函数国标检验抽样标准表免费下载红头文件格式标准下载 是其在丰富和完善人生方面所起的作用,但这种作用并不是我们最初的动机的组成部分。 47. 人们只是逐渐地才注意到机构的这—副产品,而人们把这种作用视为机构运作的指导性因素的过程则更为缓慢。 48. 虽然在与年轻人的接触中我们容易忽视自己的行为对他们的性情所产生的影响,然而在与成年人打交道时这种情况就不那么容易发生。 49. 由于我们对年轻人所做的首要工作在于使他们能够在生活中彼此相融,因此我们不禁要考虑自己是否在形成让他们获得这种能力的力量。 50. 这就使我们得以在一直讨论的广义的教育过程中进一步区分出一种更为正式的教育形式,即直接讲授或学校教育。 62 2009年考研英语真题答案 63 2009年考研英语真题答案 64 64
本文档为【2009-2013年考研英语历年真题集含答案解析】,请使用软件OFFICE或WPS软件打开。作品中的文字与图均可以修改和编辑, 图片更改请在作品中右键图片并更换,文字修改请直接点击文字进行修改,也可以新增和删除文档中的内容。
该文档来自用户分享,如有侵权行为请发邮件ishare@vip.sina.com联系网站客服,我们会及时删除。
[版权声明] 本站所有资料为用户分享产生,若发现您的权利被侵害,请联系客服邮件isharekefu@iask.cn,我们尽快处理。
本作品所展示的图片、画像、字体、音乐的版权可能需版权方额外授权,请谨慎使用。
网站提供的党政主题相关内容(国旗、国徽、党徽..)目的在于配合国家政策宣传,仅限个人学习分享使用,禁止用于任何广告和商用目的。
下载需要: 免费 已有0 人下载
最新资料
资料动态
专题动态
is_083599
暂无简介~
格式:doc
大小:346KB
软件:Word
页数:0
分类:英语六级
上传时间:2017-10-05
浏览量:34